FINAL (Includes Unit 3 - Chapters 24, 65-68)

¡Supera tus tareas y exámenes ahora con Quizwiz!

Which problem places a client at highest risk for sepsis? A. Pernicious anemia B. Pericarditis C. Post kidney transplant D. Client owns an iguana

C. Post kidney transplant The post-kidney transplant client will need to take lifelong immune suppressant therapy and is at risk for infection from internal and external organisms. Pernicious anemia is related to lack of vitamin B12, not to bone marrow failure (aplastic anemia), which would place the client at risk for infection. Inflammation of the pericardial sac is an inflammatory condition that does not pose a risk for septic shock. Although owning pets, especially cats and reptiles, poses a risk for infection, the immune-suppressed kidney transplant client has a very high risk for infection, sepsis, and death.

What are the primary pathophysiologic changes that occur in the injury or exudative phase of ARDS (SATA)? a. Atelectasis b. Shortness of breath c. Interstitial and alveolar edema d. Hyaline membranes line the alveoli e. Influx of neutrophils, monocytes, and lymphocytes

a, c, d (The injury or exudative phase is the early phase of ARDS when atelectasis and interstitial and alveoli edema occur and hyaline membranes composed of necrotic cells, protein, and fibrin line the alveoli. Together, these decrease gas exchange capability and lung compliance. Shortness of breath occurs but it is not a physiologic change. The increased inflammation and proliferation of fibroblasts occurs in the reparative or proliferative phase of ARDS, which occurs 1 to 2 weeks after the initial lung injury.)

Maintenance of fluid balance in the patient with ARDS involves a. hydration using colloids. b. administration of surfactant. c. fluid restriction and diuretics as necessary. d. keeping the hemoglobin at levels above 9 g/dL (90 g/L).

c. fluid restriction and diuretics as necessary. (In a patient with ARDS, management of fluid balance includes limiting pulmonary edema by monitoring stroke volume variation or maintaining the pulmonary artery wedge pressure as low as possible without impairing cardiac output. Patients are often placed on fluid restriction, and diuretics are used as necessary.)

A patient with a history of major depression is brought to the ED by her parents. Which of the following nursing actions is most appropriate? A) Noting that symptoms of physical illness are not relevant to the current diagnosis B) Asking the patient if she has ever thought about taking her own life C) Conducting interviews in a brief and direct manner D) Arranging for the patient to spend time alone to consider her feelings

: B Feedback: Establishing if the patient has suicidal thoughts or intents helps identify the level of depression and intervention. Physical symptoms are relevant and should be explored. Allow the patient to express feelings, and conduct the interview at a comfortable pace for the patient. Never leave the patient alone, because suicide is usually committed in solitude.

11. Which assessment finding obtained by the nurse when caring for a patient with a right radial arterial line indicates a need for the nurse to take action? a. The right hand feels cooler than the left hand. b. The mean arterial pressure (MAP) is 77 mm Hg. c. The system is delivering 3 mL of flush solution per hour. d. The flush bag and tubing were last changed 2 days previously.

A

22. A patient with respiratory failure has arterial pressure-based cardiac output (APCO) monitoring and is receiving mechanical ventilation with peak end-expiratory pressure (PEEP) of 12 cm H2O. Which information indicates that a change in the ventilator settings may be required? a. The arterial pressure is 90/46. b. The stroke volume is increased. c. The heart rate is 58 beats/minute. d. The stroke volume variation is 12%.

A

25. When evaluating a patient with a central venous catheter, the nurse observes that the insertion site is red and tender to touch and the patient's temperature is 101.8° F. What should the nurse plan to do? a. Discontinue the catheter and culture the tip. b. Use the catheter only for fluid administration. c. Change the flush system and monitor the site. d. Check the site more frequently for any swelling.

A

32. A patient who is orally intubated and receiving mechanical ventilation is anxious and is "fighting" the ventilator. Which action should the nurse take next? a. Verbally coach the patient to breathe with the ventilator. b. Sedate the patient with the ordered PRN lorazepam (Ativan). c. Manually ventilate the patient with a bag-valve-mask device. d. Increase the rate for the ordered propofol (Diprivan) infusion.

A

35. A patient who is receiving positive pressure ventilation is scheduled for a spontaneous breathing trial (SBT). Which finding by the nurse is most likely to result in postponing the SBT? a. New ST segment elevation is noted on the cardiac monitor. b. Enteral feedings are being given through an orogastric tube. c. Scattered rhonchi are heard when auscultating breath sounds. d. hydromorphone (Dilaudid) is being used to treat postoperative pain.

A

6. The purpose of adding PEEP to positive pressure ventilation is to a. increase functional residual capacity and improve oxygenation. b. increase FIO2 in an attempt to wean the patient and avoid O2 toxicity. c. determine if the patient is in synchrony with the ventilator or needs to be paralyzed. d. determine if the patient is able to be weaned and avoid the risk of pneumomediastinum.

A

A patient is admitted to the ED with suspected alcohol intoxication. The ED nurse is aware of the need to assess for conditions that can mimic acute alcohol intoxication. In light of this need, the nurse should perform what action? A) Check the patients blood glucose level. B) Assess for a documented history of major depression. C) Determine whether the patient has ingested a corrosive substance. D) Arrange for assessment of serum potassium levels.

A Feedback: Hypoglycemia can mimic alcohol intoxication and should be assessed in a patient suspected of alcohol intoxication. Potassium imbalances, depression, and poison ingestion are not noted to mimic the characteristic signs and symptoms of alcohol intoxication.

A hospitals emergency operations plan has been enacted following an industrial accident. While one nurse performs the initial triage, what should other emergency medical services personnel do? A. Perform life-saving measures. B. Classify patients according to acuity. C. Provide health promotion education. D. Modify the emergency operations plan.

A Feedback: In an emergency, patients are immediately tagged and transported or given life-saving interventions. One person performs the initial triage while other emergency medical services (EMS) personnel perform lifesaving measures and transport patients. Health promotion is not a priority during the acute stage of the crisis. Classifying patients is the task of the triage nurse. EMS personnel prioritize life-saving measures; they do not modify the operations plan.

A patient has been brought to the ED with multiple trauma after a motor vehicle accident. After immediate threats to life have been addressed, the nurse and trauma team should take what action? A) Perform a rapid physical assessment. B) Initiate health education. C) Perform diagnostic imaging. D) Establish the circumstances of the accident.

A Feedback: Once immediate threats to life have been corrected, a rapid physical examination is done to identify injuries and priorities of treatment. Health education is initiated later in the care process and diagnostic imaging would take place after a rapid physical assessment. It is not the care teams responsibility to determine the circumstances of the accident.

Which patient should the nurse prioritize as needing emergent treatment, assuming no other injuries are present except the ones outlined below? A) A patient with a blunt chest trauma with some difficulty breathing B) A patient with a sore neck who was immobilized in the field on a backboard with a cervical collar C) A patient with a possible fractured tibia with adequate pedal pulses D) A patient with an acute onset of confusion

A Feedback: The patient with blunt chest trauma possibly has a compromised airway. Establishment and maintenance of a patent airway and adequate ventilation is prioritized over other health problems, including skeletal injuries and changes in cognition.

During the primary assessment of a patient with multiple trauma, the nurse observes that the patient's right pedal pulses are absent and the leg is swollen. The nurse's first action should be to a. initiate isotonic fluid infusion through two large-bore IV lines. b. send blood to the lab for a complete blood count (CBC). c. finish the airway, breathing, circulation, disability survey. d. assess further for a cause of the decreased circulation.

A Rationale: The assessment data indicate that the patient may have arterial trauma and hemorrhage. When a possibly life-threatening injury is found during the primary survey, the nurse should immediately start interventions before proceeding with the survey. Although a CBC is indicated, administration of IV fluids should be started first. Completion of the primary survey and further assessment should be completed after the IV fluids are initiated.

The triage nurse is working in the ED. A homeless person is admitted during a blizzard with complaints of being unable to feel his feet and lower legs. Core temperature is noted at 33.2C (91.8F). The patient is intoxicated with alcohol at the time of admission and is visibly malnourished. What is the triage nurses priority in the care of this patient? A) Addressing the patients hypothermia B) Addressing the patients frostbite in his lower extremities C) Addressing the patients alcohol intoxication D) Addressing the patients malnutrition

A Feedback: The patient may also have frostbite, but hypothermia takes precedence in treatment because it is systemic rather than localized. The alcohol abuse and the alteration in nutrition do not take precedence over the treatment of hypothermia because both problems are a less acute threat to the patients survival

Emergency medical technicians arrive at the emergency department with an unresponsive client who has an oxygen mask in place. Which action should the nurse take first? a. Assess that the client is breathing adequately. b. Insert a large-bore intravenous line. c. Place the client on a cardiac monitor. d. Assess for the best neurologic response.

A The highest-priority intervention in the primary survey is to establish that the client is breathing adequately. Even though this client has an oxygen mask on, he or she may not be breathing, or may be breathing inadequately with the device in place.

A patient is in acute respiratory distress syndrome (ARDS) as a result of sepsis. Which measure would be implemented to maintain cardiac output? A Administer crystalloid fluids. B Position the patient in the Trendelenburg position. C Place the patient on fluid restriction and administer diuretics. D Perform chest physiotherapy and assist with staged coughing.

A Administer crystalloid fluids. (Low cardiac output may necessitate crystalloid fluids in addition to lowering positive end-expiratory pressure (PEEP) or administering inotropes. The Trendelenburg position (not recommended to treat hypotension) and chest physiotherapy are unlikely to relieve decreased cardiac output, and fluid restriction and diuresis would be inappropriate interventions.)

The patient has pulmonary fibrosis and experiences hypoxemia during exercise but not at rest. To plan patient care, the nurse should know the patient is experiencing which physiologic mechanism of respiratory failure? A Diffusion limitation B Intrapulmonary shunt C Alveolar hypoventilation D Ventilation-perfusion mismatch

A Diffusion limitation (The patient with pulmonary fibrosis has a thickened alveolar-capillary interface that slows gas transport, and hypoxemia is more likely during exercise than at rest. Intrapulmonary shunt occurs when alveoli fill with fluid (e.g., acute respiratory distress syndrome, pneumonia). Alveolar hypoventilation occurs when there is a generalized decrease in ventilation (e.g., restrictive lung disease, central nervous system diseases, neuromuscular diseases). Ventilation-perfusion mismatch occurs when the amount of air does not match the amount of blood that the lung receives (e.g., COPD, pulmonary embolus).)

The nurse in the cardiac care unit is caring for a patient who has developed acute respiratory failure. Which medication is used to decrease patient pulmonary congestion and agitation? A Morphine B Albuterol C Azithromycin D Methylprednisolone

A Morphine (For a patient with acute respiratory failure related to the heart, morphine is used to decrease pulmonary congestion as well as anxiety, agitation, and pain. Albuterol is used to reduce bronchospasm. Azithromycin is used for pulmonary infections. Methylprednisolone is used to reduce airway inflammation and edema.)

An emergency department (ED) nurse is preparing to transfer a client to the trauma intensive care unit. Which information should the nurse include in the nurse-to-nurse hand-off report? (SATA) a. Mechanism of injury b. Diagnostic test results c. Immunizations d. List of home medications e. Isolation precautions

A, B, E Hand-off communication should be comprehensive so that the receiving nurse can continue care for the client fluidly. Communication should be concise and should include only the most essential information for a safe transition in care. Hand-off communication should include the clients situation (reason for being in the ED), brief medical history, assessment and diagnostic findings, Transmission-Based Precautions needed, interventions provided, and response to those interventions.

2. What are the appropriate nursing interventions for the patient with delirium in the ICU (select all that apply)? a. Use clocks and calendars to maintain orientation. b. Encourage round-the-clock presence of caregivers at the bedside. c. Silence all alarms, reduce overhead paging, and avoid conversations around the patient. d. Sedate the patient with appropriate drugs to protect the patient from harmful behaviors. e. Identify physiologic factors that may be contributing to the patient's confusion and irritability.

A, D, E

The complex care provided during an emergency requires interdisciplinary collaboration. Which interdisciplinary team members are paired with the correct responsibilities? (SATA) a. Psychiatric crisis nurse Interacts with clients and families when sudden illness, serious injury, or death of a loved one may cause a crisis b. Forensic nurse examiner Performs rapid assessments to ensure clients with the highest acuity receive the quickest evaluation, treatment, and prioritization of resources c. Triage nurse Provides basic life support interventions such as oxygen, basic wound care, splinting, spinal immobilization, and monitoring of vital signs d. Emergency medical technician Obtains client histories, collects evidence, and offers counseling and follow-up care for victims of rape, child abuse, and domestic violence e. Paramedic Provides prehospital advanced life support, including cardiac monitoring, advanced airway management, and medication administration

A, E The psychiatric crisis nurse evaluates clients with emotional behaviors or mental illness and facilitates follow-up treatment plans. The psychiatric crisis nurse also works with clients and families when experiencing a crisis. Paramedics are advanced life support providers who can perform advanced techniques that may include cardiac monitoring, advanced airway management and intubation, establishing IV access, and administering drugs en route to the emergency department. The forensic nurse examiner is trained to recognize evidence of abuse and to intervene on the clients behalf. The forensic nurse examiner will obtain client histories, collect evidence, and offer counseling and follow-up care for victims of rape, child abuse, and domestic violence. The triage nurse performs rapid assessments to ensure clients with the highest acuity receive the quickest evaluation, treatment, and prioritization of resources. The emergency medical technician is usually the first caregiver and provides basic life support and transportation to the emergency department.

A client is exhibiting signs and symptoms of early shock. What is important for the nurse to do to support the psychosocial integrity of the client? (Select all that apply.) A. Ask family members to stay with the client. B. Call the health care provider. C. Increase IV and oxygen rates. D. Remain with the client. E. Reassure the client that everything is being done for him or her.

A. Ask family members to stay with the client. D. Remain with the client. E. Reassure the client that everything is being done for him or her. Having a familiar person nearby may provide comfort to the client. The nurse should remain with the client who is demonstrating physiologic deterioration. Offering genuine reassurance supports the client who is anxious. The health care provider should be notified, and increasing IV and oxygen rates may be needed, but these actions do not support the client's psychosocial integrity.

A client is admitted to the hospital with two of the systemic inflammatory response syndrome variables: temperature of 95° F (35° C) and high white blood cell count. Which intervention from the sepsis resuscitation bundle does the nurse initiate? A. Broad-spectrum antibiotics B. Blood transfusion C. Cooling baths D. NPO status

A. Broad-spectrum antibiotics Broad-spectrum antibiotics must be initiated within 1 hour of establishing diagnosis. A blood transfusion is indicated for low red blood cell count or low hemoglobin and hematocrit; transfusion is not part of the sepsis resuscitation bundle. Cooling baths are not indicated because the client is hypothermic, nor is this part of the sepsis resuscitation bundle. NPO status is not indicated for this client, nor is it part of the sepsis resuscitation bundle.

The nursing assistant is concerned about a postoperative client with blood pressure (BP) of 90/60 mm Hg, heart rate of 80 beats/min, and respirations of 22 breaths/min. What does the supervising nurse do? A. Compare these vital signs with the last several readings. B. Request that the surgeon see the client. C. Increase the rate of intravenous fluids. D. Reassess vital signs using different equipment.

A. Compare these vital signs with the last several readings. Vital sign trends must be taken into consideration; a BP of 90/60 mm Hg may be normal for this client. Calling the surgeon is not necessary at this point, and increasing IV fluids is not indicated. The same equipment should be used when vital signs are taken postoperatively.

The client with which problem is at highest risk for hypovolemic shock? A. Esophageal varices B. Kidney failure C. Arthritis and daily acetaminophen use D. Kidney stone

A. Esophageal varices Esophageal varices are caused by portal hypertension; the portal vessels are under high pressure and are prone to rupture, causing massive upper gastrointestinal tract bleeding and hypovolemic shock. As the kidneys fail, fluid is typically retained, causing fluid volume excess, not hypovolemia. Nonsteroidal anti-inflammatory drugs such as naproxen and ibuprofen, not acetaminophen, predispose the client to gastrointestinal bleeding and hypovolemia. Although a kidney stone may cause hematuria, there is not generally massive blood loss or hypovolemia.

The client with which laboratory result is at risk for hemorrhagic shock? A. International normalized ratio (INR) 7.9 B. Partial thromboplastin time (PTT) 12.5 seconds C. Platelets 170,000/mm3 D. Hemoglobin 8.2 g/dL

A. International normalized ratio (INR) 7.9 Prolonged INR indicates that blood takes longer than normal to clot; this client is at risk for bleeding. PTT of 12.5 seconds and a platelet value of 170,000/mm3 are both normal and pose no risk for bleeding. Although a hemoglobin of 8.2 g/dL is low, the client could have severe iron deficiency or could have received medication affecting the bone marrow.

The nurse reviews the medical record of a client with hemorrhagic shock, which contains the following information: Pulse 140 beats/min and thready, ABG respiratory acidosis, Blood pressure 60/40 mm Hg, Lactate level 7 mOsm/L, Respirations 40/min and shallow. All of these provider prescriptions are given for the client. Which does the nurse carry out first? A. Notify anesthesia for endotracheal intubation. B. Give Plasmanate 1 unit now. C. Give normal saline solution 250 mL/hr. D. Type and crossmatch for 4 units of packed red blood cells (PRBCs).

A. Notify anesthesia for endotracheal intubation. Establishing an airway is the priority in all emergency situations. Although administering Plasmanate and normal saline, and typing and crossmatching for 4 units of PRBCs are important actions, airway always takes priority.

Which clients are at immediate risk for hypovolemic shock? (Select all that apply.) A. Unrestrained client in motor vehicle accident B. Construction worker C. Athlete D. Surgical intensive care client E. 85-year-old with gastrointestinal virus

A. Unrestrained client in motor vehicle accident D. Surgical intensive care client E. 85-year-old with gastrointestinal virus The client who is unrestrained in a motor vehicle accident is prone to multiple trauma and bleeding. Surgical clients are at high risk for hypovolemic shock owing to fluid loss and hemorrhage. Older adult clients are prone to shock; a gastrointestinal virus results in fluid losses. Unless injured or working in excessive heat, the construction worker and the athlete are not at risk for hypovolemic shock; they may be at risk for dehydration.

The nurse estimates the extent of a burn using the rule of nines for a patient who has been admitted with deep partial-thickness burns of the anterior trunk and the entire left arm. What percentage of the patient's total body surface area (TBSA) has been injured?

ANS: 27% When using the rule of nines, the anterior trunk is considered to cover 18% of the patient's body and the anterior (4.5%) and posterior (4.5%) left arm equals 9%. DIF: Cognitive Level: Understand (comprehension)

An 80-kg patient with burns over 30% of total body surface area (TBSA) is admitted to the burn unit. Using the Parkland formula of 4 mL/kg/%TBSA, what is the IV infusion rate (mL/hour) for lactated Ringer's solution that the nurse will give during the first 8 hours?

ANS: 600 mL The Parkland formula states that patients should receive 4 mL/kg/%TBSA burned during the first 24 hours. Half of the total volume is given in the first 8 hours and then the remaining half is given over 16 hours: 4 80 30 = 9600 mL total volume; 9600/2 = 4800 mL in the first 8 hours; 4800 mL/8 hr = 600 mL/hr. DIF: Cognitive Level: Apply (application)

The following four patients arrive in the emergency department (ED) after a motor vehicle collision. In which order should the nurse assess them? (Put a comma and a space between each answer choice [A, B, C, D, E].) a. A 74-yr-old patient with palpitations and chest pain b. A 43-yr-old patient complaining of 7/10 abdominal pain c. A 21-yr-old patient with multiple fractures of the face and jaw d. A 37-yr-old patient with a misaligned lower left leg with intact pulses

ANS: C, A, B, D The highest priority is to assess the 21-yr-old patient for airway obstruction, which is the most life-threatening injury. The 74-yr-old patient may have chest pain from cardiac ischemia and should be assessed and have diagnostic testing for this pain. The 43-yr-old patient may have abdominal trauma or bleeding and should be seen next to assess circulatory status. The 37-yr-old patient appears to have a possible fracture of the left leg and should be seen soon, but this patient has the least life-threatening injury. DIF: Cognitive Level: Analyze (analysis)

In which order will the nurse take these actions when doing a dressing change for a partial-thickness burn wound on a patient's chest? (Put a comma and a space between each answer choice [A, B, C, D, E].) a. Apply sterile gauze dressing. b. Document wound appearance. c. Apply silver sulfadiazine cream. d. Give IV fentanyl (Sublimaze). e. Clean wound with saline-soaked gauze.

ANS: D, E, C, A, B Because partial-thickness burns are very painful, the nurse's first action should be to give pain medications. The wound will then be cleaned, antibacterial cream applied, and covered with a new sterile dressing. The last action should be to document the appearance of the wound. DIF: Cognitive Level: Analyze (analysis)

A patient arrives in the emergency department with facial and chest burns caused by a house fire. Which action should the nurse take first? a. Auscultate the patient's lung sounds. b. Determine the extent and depth of the burns. c. Give the prescribed hydromorphone (Dilaudid). d. Infuse the prescribed lactated Ringer's solution.

ANS: A A patient with facial and chest burns is at risk for inhalation injury and assessment of airway and breathing is the priority. The other actions will be completed after airway management is assured. DIF: Cognitive Level: Analyze (analysis)

Norepinephrine (Levophed) has been prescribed for a patient who was admitted with dehydration and hypotension. Which patient data indicate that the nurse should consult with the health care provider before starting the norepinephrine? a. The patient's central venous pressure is 3 mm Hg. b. The patient is in sinus tachycardia at 120 beats/min. c. The patient is receiving low dose dopamine (Intropin). d. The patient has had no urine output since being admitted.

ANS: A Adequate fluid administration is essential before administration of vasopressors to patients with hypovolemic shock. The patient's low central venous pressure indicates a need for more volume replacement. The other patient data are not contraindications to norepinephrine administration.

Which information about a patient who is receiving cisatracurium (Nimbex) to prevent asynchronous breathing with the positive pressure ventilator requires immediate action by the nurse? a. Only continuous IV opioids have been ordered. b. The patient does not respond to verbal stimulation. c. There is no cough or gag when the patient is suctioned. d. The patient's oxygen saturation fluctuates between 90% to 93%.

ANS: A Because neuromuscular blockade is extremely anxiety provoking, it is essential that patients who are receiving neuromuscular blockade receive concurrent sedation and analgesia. Absence of response to stimuli is expected in patients receiving neuromuscular blockade. The oxygen saturation is adequate.

A patient with septic shock has a BP of 70/46 mm Hg, pulse 136, respirations 32, temperature 104° F, and blood glucose 246 mg/dL. Which intervention ordered by the health care provider should the nurse implement first? a. Give normal saline IV at 500 mL/hr. b. Give acetaminophen (Tylenol) 650 mg rectally. c. Start insulin drip to maintain blood glucose at 110 to 150 mg/dL. d. Start norepinephrine (Levophed) to keep systolic blood pressure >90 mm Hg.

ANS: A Because of the low systemic vascular resistance (SVR) associated with septic shock, fluid resuscitation is the initial therapy. The other actions also are appropriate, and should be initiated quickly as well.

A 78-kg patient with septic shock has a urine output of 30 mL/hr for the past 3 hours. The pulse rate is 120/minute and the central venous pressure and pulmonary artery wedge pressure are low. Which order by the health care provider will the nurse question? a. Give PRN furosemide (Lasix) 40 mg IV. b. Increase normal saline infusion to 250 mL/hr. c. Administer hydrocortisone (Solu-Cortef) 100 mg IV. d. Titrate norepinephrine (Levophed) to keep systolic BP >90 mm Hg.

ANS: A Furosemide will lower the filling pressures and renal perfusion further for the patient with septic shock. The other orders are appropriate.

While caring for a patient who has been admitted with a pulmonary embolism, the nurse notes a change in the patient's oxygen saturation (SpO2) from 94% to 88%. Which action should the nurse take next? a. Increase the oxygen flow rate. b. Suction the patient's oropharynx. c. Instruct the patient to cough and deep breathe. d. Help the patient to sit in a more upright position.

ANS: A Increasing oxygen flow rate will usually improve oxygen saturation in patients with ventilation-perfusion mismatch, as occurs with pulmonary embolism. Because the problem is with perfusion, actions that improve ventilation, such as deep breathing and coughing, sitting upright, and suctioning, are not likely to improve oxygenation.

12. A patient arrives in the emergency department (ED) several hours after taking "25 to 30" acetaminophen (Tylenol) tablets. Which action will the nurse plan to take? a. Give N-acetylcysteine. b. Discuss the use of chelation therapy. c. Start oxygen using a non-rebreather mask. d. Have the patient drink large amounts of water.

ANS: A N-acetylcysteine is the recommended treatment to prevent liver damage after acetaminophen overdose. The other actions might be used for other types of poisoning, but they will not be appropriate for a patient with acetaminophen poisoning.

The charge nurse observes the following actions being taken by a new nurse on the burn unit. Which action by the new nurse would require immediate intervention by the charge nurse? a. The new nurse uses clean gloves when applying antibacterial cream to a burn wound. b. The new nurse obtains burn cultures when the patient has a temperature of 95.2° F (35.1° C). c. The new nurse gives PRN fentanyl (Sublimaze) IV to a patient 5 minutes before a dressing change. d. The new nurse calls the health care provider when a nondiabetic patient's serum glucose is elevated.

ANS: A Sterile gloves should be worn when applying medications or dressings to a burn. Hypothermia is an indicator of possible sepsis, and cultures are appropriate. Nondiabetic patients may require insulin because stress and high calorie intake may lead to temporary hyperglycemia. Fentanyl peaks 5 minutes after IV administration and should be used just before and during dressing changes for pain management. DIF: Cognitive Level: Apply (application)

During change-of-shift report, the nurse is told that a patient has been admitted with dehydration and hypotension after having vomiting and diarrhea for 4 days. Which finding is most important for the nurse to report to the health care provider? a. New onset of confusion b. Heart rate 112 beats/minute c. Decreased bowel sounds d. Pale, cool, and dry extremities

ANS: A The changes in mental status are indicative that the patient is in the progressive stage of shock and that rapid intervention is needed to prevent further deterioration. The other information is consistent with compensatory shock

Which data collected by the nurse caring for a patient who has cardiogenic shock indicate that the patient may be developing multiple organ dysfunction syndrome (MODS)? a. The patient's serum creatinine level is elevated. b. The patient complains of intermittent chest pressure. c. The patient's extremities are cool and pulses are weak. d. The patient has bilateral crackles throughout lung fields.

ANS: A The elevated serum creatinine level indicates that the patient has renal failure as well as heart failure. The crackles, chest pressure, and cool extremities are all consistent with the patient's diagnosis of cardiogenic shock.

A patient is admitted to the emergency department (ED) for shock of unknown etiology. The first action by the nurse should be to a. administer oxygen. b. obtain a 12-lead electrocardiogram (ECG). c. obtain the blood pressure. d. check the level of consciousness.

ANS: A The initial actions of the nurse are focused on the ABCs—airway, breathing, and circulation—and administration of oxygen should be done first. The other actions should be accomplished as rapidly as possible after oxygen administration.

A nurse is caring for an obese patient with right lower lobe pneumonia. Which position will be best to improve gas exchange? a. On the left side b. On the right side c. In the tripod position d. In the high-Fowler's position

ANS: A The patient should be positioned with the "good" lung in the dependent position to improve the match between ventilation and perfusion. The obese patient's abdomen will limit respiratory excursion when sitting in the high-Fowler's or tripod positions.

A nurse is caring for a patient with shock of unknown etiology whose hemodynamic monitoring indicates BP 92/54, pulse 64, and an elevated pulmonary artery wedge pressure. Which collaborative intervention ordered by the health care provider should the nurse question? a. Infuse normal saline at 250 mL/hr. b. Keep head of bed elevated to 30 degrees. c. Hold nitroprusside (Nipride) if systolic BP <90 mm Hg. d. Titrate dobutamine (Dobutrex) to keep systolic BP >90 mm Hg.

ANS: A The patient's elevated pulmonary artery wedge pressure indicates volume excess. A saline infusion at 250 mL/hr will exacerbate the volume excess. The other actions are appropriate for the patient.

DELTE13. A triage nurse in a busy emergency department (ED) assesses a patient who complains of 7/10 abdominal pain and states, "I had a temperature of 103.9° F (39.9° C) at home." The nurse's first action should be to a. assess the patient's current vital signs. b. give acetaminophen (Tylenol) per agency protocol. c. ask the patient to provide a clean-catch urine for urinalysis. d. tell the patient that it will be 1 to 2 hours before seeing a health care provider.

ANS: A The patient's pain and statement about an elevated temperature indicate that the nurse should obtain vital signs before deciding how rapidly the patient should be seen by the health care provider. A urinalysis may be appropriate, but this would be done after the vital signs are taken. The nurse will not give acetaminophen before confirming a current temperature elevation.

A triage nurse in a busy emergency department (ED) assesses a patient who complains of 7/10 abdominal pain and states, "I had a temperature of 103.9° F (39.9° C) at home." The nurse's first action should be to a. assess the patient's current vital signs. b. give acetaminophen (Tylenol) per agency protocol. c. ask the patient to provide a clean-catch urine for urinalysis. d. tell the patient that it will be 1 to 2 hours before seeing a health care provider.

ANS: A The patient's pain and statement about an elevated temperature indicate that the nurse should obtain vital signs before deciding how rapidly the patient should be seen by the health care provider. A urinalysis may be appropriate, but this would be done after the vital signs are taken. The nurse will not give acetaminophen before confirming a current temperature elevation. DIF: Cognitive Level: Analyze (analysis)

The nurse reviews the electronic medical record for a patient scheduled for a total hip replacement. Which assessment data shown in the accompanying figure increase the patient's risk for respiratory complications after surgery? a. Albumin level and recent weight loss b. Mild confusion and recent weight loss c. Age and recent arthroscopic procedure. d. Anemia and recent arthroscopic procedure

ANS: A The patient's recent weight loss and low protein stores indicate possible muscle weakness, which make it more difficult for an older patient to recover from the effects of general anesthesia and immobility associated with the hip surgery. The other information will also be noted by the nurse but does not place the patient at higher risk for respiratory failure.

20. Following an earthquake, patients are triaged by emergency medical personnel and transported to the emergency department (ED). Which patient will the nurse need to assess first? a. A patient with a red tag b. A patient with a blue tag c. A patient with a black tag d. A patient with a yellow tag

ANS: A The red tag indicates a patient with a life-threatening injury requiring rapid treatment. The other tags indicate patients with less urgent injuries or those who are likely to die.

3. After the return of spontaneous circulation following the resuscitation of a patient who had a cardiac arrest, therapeutic hypothermia is ordered. Which action will the nurse include in the plan of care? a. Initiate cooling per protocol. b. Avoid the use of sedative drugs. c. Check mental status every 15 minutes. d. Rewarm if temperature is below 91° F (32.8° C).

ANS: A When therapeutic hypothermia is used postresuscitation, external cooling devices or cold normal saline infusions are used to rapidly lower body temperature to 89.6° F to 93.2° F (32° C to 34° C). Because hypothermia will decrease brain activity, assessing mental status every 15 minutes is not done at this stage. Sedative drugs are given during therapeutic hypothermia.

An employee spills industrial acids on both arms and legs at work. What action should the occupational health nurse take first? a. Remove nonadherent clothing and wristwatch. b. Apply an alkaline solution to the affected area. c. Place a cool compress on the area of exposure. d. Cover the affected area with dry, sterile dressings.

ANS: A With chemical burns, the initial action is to remove the chemical from contact with the skin as quickly as possible. Remove nonadherent clothing, shoes, watches, jewelry, glasses, or contact lenses (if the face was exposed). Flush the chemical from the wound and surrounding area with copious amounts of saline solution or water. Covering the affected area or placing cool compresses on the area will leave the chemical in contact with the skin. Application of an alkaline solution is not recommended. DIF: Cognitive Level: Apply (application)

Which preventive actions by the nurse will help limit the development of systemic inflammatory response syndrome (SIRS) in patients admitted to the hospital (select all that apply)? a. Use aseptic technique when caring for invasive lines or devices. b. Ambulate postoperative patients as soon as possible after surgery. c. Remove indwelling urinary catheters as soon as possible after surgery. d. Advocate for parenteral nutrition for patients who cannot take oral feedings. e. Administer prescribed antibiotics within 1 hour for patients with possible sepsis.

ANS: A, B, C, E Because sepsis is the most frequent etiology for SIRS, measures to avoid infection such as removing indwelling urinary catheters as soon as possible, use of aseptic technique, and early ambulation should be included in the plan of care. Adequate nutrition is important in preventing SIRS. Enteral, rather than parenteral, nutrition is preferred when patients are unable to take oral feedings because enteral nutrition helps maintain the integrity of the intestine, thus decreasing infection risk. Antibiotics should be administered within 1 hour after being prescribed to decrease the risk of sepsis progressing to SIRS.

A patient with suspected neurogenic shock after a diving accident has arrived in the emergency department. A cervical collar is in place. Which actions should the nurse take (select all that apply)? a. Prepare to administer atropine IV. b. Obtain baseline body temperature. c. Infuse large volumes of lactated Ringer's solution. d. Provide high-flow oxygen (100%) by non-rebreather mask. e. Prepare for emergent intubation and mechanical ventilation.

ANS: A, B, D, E All of the actions are appropriate except to give large volumes of lactated Ringer's solution. The patient with neurogenic shock usually has a normal blood volume, and it is important not to volume overload the patient. In addition, lactated Ringer's solution is used cautiously in all shock situations because the failing liver cannot convert lactate to bicarbonate.

A patient with extensive electrical burn injuries is admitted to the emergency department. Which prescribed intervention should the nurse implement first? a. Assess pain level. c. Check potassium level. b. Place on heart monitor. d. Assess oral temperature.

ANS: B After an electrical burn, the patient is at risk for life-threatening dysrhythmias and should be placed on a heart monitor. Assessing the oral temperature and pain is not as important as assessing for cardiac dysrhythmias. Checking the potassium level is important, but it will take time before the laboratory results are back. The first intervention is to place the patient on a heart monitor and assess for dysrhythmias so that they can be monitored and treated if necessary. DIF: Cognitive Level: Analyze (analysis)

After change-of-shift report in the progressive care unit, who should the nurse care for first? a. Patient who had an inferior myocardial infarction 2 days ago and has crackles in the lung bases b. Patient with suspected urosepsis who has new orders for urine and blood cultures and antibiotics c. Patient who had a T5 spinal cord injury 1 week ago and currently has a heart rate of 54 beats/minute d. Patient admitted with anaphylaxis 3 hours ago who now has clear lung sounds and a blood pressure of 108/58 mm Hg

ANS: B Antibiotics should be administered within the first hour for patients who have sepsis or suspected sepsis in order to prevent progression to systemic inflammatory response syndrome (SIRS) and septic shock. The data on the other patients indicate that they are more stable. Crackles heard only at the lung bases do not require immediate intervention in a patient who has had a myocardial infarction. Mild bradycardia does not usually require atropine in patients who have a spinal cord injury. The findings for the patient admitted with anaphylaxis indicate resolution of bronchospasm and hypotension.

Which finding is the best indicator that the fluid resuscitation for a patient with hypovolemic shock has been effective? a. Hemoglobin is within normal limits. b. Urine output is 60 mL over the last hour. c. Central venous pressure (CVP) is normal. d. Mean arterial pressure (MAP) is 72 mm Hg.

ANS: B Assessment of end organ perfusion, such as an adequate urine output, is the best indicator that fluid resuscitation has been successful. The hemoglobin level, CVP, and MAP are useful in determining the effects of fluid administration, but they are not as useful as data indicating good organ perfusion.

The nurse is caring for a patient who has septic shock. Which assessment finding is most important for the nurse to report to the health care provider? a. Blood pressure (BP) 92/56 mm Hg b. Skin cool and clammy c. Oxygen saturation 92% d. Heart rate 118 beats/minute

ANS: B Because patients in the early stage of septic shock have warm and dry skin, the patient's cool and clammy skin indicates that shock is progressing. The other information will also be reported, but does not indicate deterioration of the patient's status.

A patient has just been admitted with a 40% total body surface area (TBSA) burn injury. To maintain adequate nutrition, the nurse should plan to take which action? a. Administer vitamins and minerals intravenously. b. Insert a feeding tube and initiate enteral feedings. c. Infuse total parenteral nutrition via a central catheter. d. Encourage an oral intake of at least 5000 kcal per day.

ANS: B Enteral feedings can usually be started during the emergent phase at low rates and increased over 24 to 48 hours to the goal rate. During the emergent phase, the patient will be unable to eat enough calories to meet nutritional needs and may have a paralytic ileus that prevents adequate nutrient absorption. Vitamins and minerals may be administered during the emergent phase, but these will not assist in meeting the patient's caloric needs. Parenteral nutrition increases the infection risk, does not help preserve gastrointestinal function, and is not routinely used in burn patients unless the gastrointestinal tract is not available for use. DIF: Cognitive Level: Apply (application)

The following interventions are ordered by the health care provider for a patient who has respiratory distress and syncope after eating strawberries. Which will the nurse complete first? a. Start a normal saline infusion. b. Give epinephrine (Adrenalin). c. Start continuous ECG monitoring. d. Give diphenhydramine (Benadryl).

ANS: B Epinephrine rapidly causes peripheral vasoconstriction, dilates the bronchi, and blocks the effects of histamine and reverses the vasodilation, bronchoconstriction, and histamine release that cause the symptoms of anaphylaxis. The other interventions are also appropriate but would not be the first ones completed.

1. During the primary assessment of a victim of a motor vehicle collision, the nurse determines that the patient has an unobstructed airway. Which action should the nurse take next? a. Palpate extremities for bilateral pulses. b. Observe the patient's respiratory effort. c. Check the patient's level of consciousness. d. Examine the patient for any external bleeding.

ANS: B Even with a patent airway, patients can have other problems that compromise ventilation, so the next action is to assess the patient's breathing. The other actions are also part of the initial survey but assessment of breathing should be done immediately after assessing for airway patency.

During the primary assessment of a victim of a motor vehicle collision, the nurse determines that the patient has an unobstructed airway. Which action should the nurse take next? a. Palpate extremities for bilateral pulses. b. Observe the patient's respiratory effort. c. Check the patient's level of consciousness. d. Examine the patient for any external bleeding.

ANS: B Even with a patent airway, patients can have other problems that compromise ventilation, so the next action is to assess the patient's breathing. The other actions are also part of the initial survey but assessment of breathing should be done immediately after assessing for airway patency. DIF: Cognitive Level: Apply (application)

6. A patient who has experienced blunt abdominal trauma during a motor vehicle collision is complaining of increasing abdominal pain. The nurse will plan to teach the patient about the purpose of a. peritoneal lavage. b. abdominal ultrasonography. c. nasogastric (NG) tube placement. d. magnetic resonance imaging (MRI).

ANS: B For patients who are at risk for intraabdominal bleeding, focused abdominal ultrasonography is the preferred method to assess for intraperitoneal bleeding. An MRI would not be used. Peritoneal lavage is an alternative, but it is more invasive. An NG tube would not be helpful in the diagnosis of intraabdominal bleeding.

Gastric lavage and administration of activated charcoal are ordered for an unconscious patient who has been admitted to the emergency department (ED) after ingesting 30 lorazepam (Ativan) tablets. Which prescribed action should the nurse plan to do first? a. Insert a large-bore orogastric tube. b. Assist with intubation of the patient. c. Prepare a 60-mL syringe with saline. d. Give first dose of activated charcoal.

ANS: B In an unresponsive patient, intubation is done before gastric lavage and activated charcoal administration to prevent aspiration. The other actions will be implemented after intubation. DIF: Cognitive Level: Analyze (analysis)

Which intervention will the nurse include in the plan of care for a patient who has cardiogenic shock? a. Check temperature every 2 hours. b. Monitor breath sounds frequently. c. Maintain patient in supine position. d. Assess skin for flushing and itching.

ANS: B Since pulmonary congestion and dyspnea are characteristics of cardiogenic shock, the nurse should assess the breath sounds frequently. The head of the bed is usually elevated to decrease dyspnea in patients with cardiogenic shock. Elevated temperature and flushing or itching of the skin are not typical of cardiogenic shock.

A patient who is unconscious after a fall from a ladder is transported to the emergency department by emergency medical personnel. During the primary survey of the patient, the nurse should a. obtain a complete set of vital signs. b. obtain a Glasgow Coma Scale score. c. attach an electrocardiogram monitor. d. ask about chronic medical conditions.

ANS: B The Glasgow Coma Scale is included when assessing for disability during the primary survey. The other information is part of the secondary survey. DIF: Cognitive Level: Apply (application)

A patient with cardiogenic shock has the following vital signs: BP 102/50, pulse 128, respirations 28. The pulmonary artery wedge pressure (PAWP) is increased and cardiac output is low. The nurse will anticipate an order for which medication? a. 5% human albumin b. Furosemide (Lasix) IV c. Epinephrine (Adrenalin) drip d. Hydrocortisone (Solu-Cortef)

ANS: B The PAWP indicates that the patient's preload is elevated, and furosemide is indicated to reduce the preload and improve cardiac output. Epinephrine would further increase heart rate and myocardial oxygen demand. 5% human albumin would also increase the PAWP. Hydrocortisone might be considered for septic or anaphylactic shock.

A patient with circumferential burns of both legs develops a decrease in dorsalis pedis pulse strength and numbness in the toes. Which action should the nurse take first? a. Monitor the pulses every hour. b. Notify the health care provider. c. Elevate both legs above heart level with pillows. d. Encourage the patient to flex and extend the toes.

ANS: B The decrease in pulse and numbness in a patient with circumferential burns indicates decreased circulation to the legs and the need for an escharotomy. Monitoring the pulses is not an adequate response to the decrease in circulation. Elevating the legs or increasing toe movement will not improve the patient's circulation. DIF: Cognitive Level: Apply (application)

The oxygen saturation (SpO2) for a patient with left lower lobe pneumonia is 90%. The patient has rhonchi, a weak cough effort, and complains of fatigue. Which action is a priority for the nurse to take? a. Position the patient on the left side. b. Assist the patient with staged coughing. c. Place a humidifier in the patient's room. d. Schedule a 2-hour rest period for the patient.

ANS: B The patient's assessment indicates that assisted coughing is needed to help remove secretions, which will improve oxygenation. A 2-hour rest period at this time may allow the oxygen saturation to drop further. Humidification will not be helpful unless the secretions can be mobilized. Positioning on the left side may cause a further decrease in oxygen saturation because perfusion will be directed more toward the more poorly ventilated lung.

A patient is admitted to the burn unit with burns to the head, face, and hands. Initially, wheezes are heard, but an hour later, the lung sounds are decreased and no wheezes are audible. What is the best action for the nurse to take? a. Encourage the patient to cough and auscultate the lungs again. b. Notify the health care provider and prepare for endotracheal intubation. c. Document the results and continue to monitor the patient's respiratory rate. d. Reposition the patient in high-Fowler's position and reassess breath sounds.

ANS: B The patient's history and clinical manifestations suggest airway edema, and the health care provider should be notified immediately so that intubation can be done rapidly. Placing the patient in a more upright position or having the patient cough will not address the problem of airway edema. Continuing to monitor is inappropriate because immediate action should occur. DIF: Cognitive Level: Apply (application)

A patient with respiratory failure has a respiratory rate of 6 breaths/minute and an oxygen saturation (SpO2) of 88%. The patient is increasingly lethargic. Which intervention will the nurse anticipate? a. Administration of 100% oxygen by non-rebreather mask b. Endotracheal intubation and positive pressure ventilation c. Insertion of a mini-tracheostomy with frequent suctioning d. Initiation of continuous positive pressure ventilation (CPAP)

ANS: B The patient's lethargy, low respiratory rate, and SpO2 indicate the need for mechanical ventilation with ventilator-controlled respiratory rate. Administration of high flow oxygen will not be helpful because the patient's respiratory rate is so low. Insertion of a mini-tracheostomy will facilitate removal of secretions, but it will not improve the patient's respiratory rate or oxygenation. CPAP requires that the patient initiate an adequate respiratory rate to allow adequate gas exchange.

The nurse caring for a patient admitted with burns over 30% of the body surface assesses that urine output has dramatically increased. Which action by the nurse would best support maintaining kidney function? a. Monitor white blood cells (WBCs). b. Continue to measure the urine output. c. Assess that blisters and edema have subsided. d. Encourage the patient to eat an adequate number of calories.

ANS: B The patient's urine output indicates that the patient is entering the acute phase of the burn injury and moving on from the emergent stage. At the end of the emergent phase, capillary permeability normalizes, and the patient begins to diurese large amounts of urine with a low specific gravity. Although this may occur at about 48 hours, it may be longer in some patients. Blisters and edema begin to resolve, but this process requires more time. WBCs may increase or decrease, based on the patient's immune status and any infectious processes. The WBC count does not indicate kidney function. Although adequate nutrition is important for healing, it does not ensure adequate kidney functioning. DIF: Cognitive Level: Understand (comprehension)

A nurse is caring for a patient who has burns of the ears, head, neck, and right arm and hand. The nurse should place the patient in which position? a. Place the right arm and hand flexed in a position of comfort. b. Elevate the right arm and hand on pillows and extend the fingers. c. Assist the patient to a supine position with a small pillow under the head. d. Position the patient in a side-lying position with rolled towel under the neck.

ANS: B The right hand and arm should be elevated to reduce swelling and the fingers extended to avoid flexion contractures (even though this position may not be comfortable for the patient). The patient with burns of the ears should not use a pillow for the head because this will put pressure on the ears, and the pillow may stick to the ears. Patients with neck burns should not use a pillow or rolled towel because the head should be maintained in an extended position in order to avoid contractures. DIF: Cognitive Level: Apply (application)

Eight hours after a thermal burn covering 50% of a patient's total body surface area (TBSA), the nurse assesses the patient. The patient weighs 92 kg (202.4 lb). Which information would be a priority to communicate to the health care provider? a. Blood pressure is 95/48 per arterial line. b. Urine output of 41 mL over past 2 hours. c. Serous exudate is leaking from the burns. d. Heart monitor shows sinus tachycardia of 108.

ANS: B The urine output should be at least 0.5 to 1.0 mL/kg/hr during the emergent phase, when the patient is at great risk for hypovolemic shock. The nurse should notify the health care provider because a higher IV fluid rate is needed. BP during the emergent phase should be greater than 90 mm Hg systolic and the pulse rate should be less than 120 beats/min. Serous exudate from the burns is expected during the emergent phase. DIF: Cognitive Level: Analyze (analysis)

Which patient should the nurse assess first? a. A patient with burns who is complaining of level 8 (0 to 10 scale) pain b. A patient with smoke inhalation who has wheezes and altered mental status c. A patient with full-thickness leg burns who is scheduled for a dressing change d. A patient with partial thickness burns who is receiving IV fluids at 500 mL/hr

ANS: B This patient has evidence of lower airway injury and hypoxemia, and should be assessed immediately to determine the need for O2 or intubation (or both). The other patients should also be assessed as rapidly as possible, but they do not have evidence of life-threatening complications. DIF: Cognitive Level: Analyze (analysis)

A young adult patient who is in the rehabilitation phase 6 months after a severe face and neck burn tells the nurse, "I'm sorry that I'm still alive. My life will never be normal again." Which response by the nurse is best? a. "Most people recover after a burn and feel satisfied with their lives." b. "It's true that your life may be different. What concerns you the most?" c. "Why do you feel that way? It will get better as your recovery progresses." d. "It is really too early to know how much your life will be changed by the burn."

ANS: B This response acknowledges the patient's feelings and asks for more assessment data that will help in developing an appropriate plan of care to assist the patient with the emotional response to the burn injury. The other statements are accurate but do not acknowledge the anxiety and depression that the patient is expressing. DIF: Cognitive Level: Apply (application)

While the patient's full-thickness burn wounds to the face are exposed, what nursing action prevents cross contamination? a. Use sterile gloves when removing dressings. b. Wear gown, cap, mask, and gloves during care. c. Keep the room temperature at 70° F (20° C) at all times. d. Give IV antibiotics to prevent bacterial colonization of wounds.

ANS: B Use of gowns, caps, masks, and gloves during all patient care will decrease the possibility of wound contamination for a patient whose burns are not covered. When removing contaminated dressings and washing the dirty wound, use nonsterile, disposable gloves. The room temperature should be kept at approximately 85° F for patients with open burn wounds to prevent shivering. Systemic antibiotics are not well absorbed into deep burns because of the lack of circulation. DIF: Cognitive Level: Apply (application)

After receiving 2 L of normal saline, the central venous pressure for a patient who has septic shock is 10 mm Hg, but the blood pressure is still 82/40 mm Hg. The nurse will anticipate an order for a. nitroglycerine (Tridil). b. norepinephrine (Levophed). c. sodium nitroprusside (Nipride). d. methylprednisolone (Solu-Medrol).

ANS: B When fluid resuscitation is unsuccessful, vasopressor drugs are administered to increase the systemic vascular resistance (SVR) and blood pressure, and improve tissue perfusion. Nitroglycerin would decrease the preload and further drop cardiac output and BP. Methylprednisolone (Solu-Medrol) is considered if blood pressure does not respond first to fluids and vasopressors. Nitroprusside is an arterial vasodilator and would further decrease SVR.

When assessing a patient who spilled hot oil on the right leg and foot, the nurse notes dry, pale, and hard skin. The patient states that the burn is not painful. What term would the nurse use to document the burn depth? a. First-degree skin destruction b. Full-thickness skin destruction c. Deep partial-thickness skin destruction d. Superficial partial-thickness skin destruction

ANS: B With full-thickness skin destruction, the appearance is pale and dry or leathery, and the area is painless because of the associated nerve destruction. Erythema, swelling, and blisters point to a deep partial-thickness burn. With superficial partial-thickness burns, the area is red, but no blisters are present. First-degree burns exhibit erythema, blanching, and pain. DIF: Cognitive Level: Understand (comprehension)

A patient with burns covering 40% total body surface area (TBSA) is in the acute phase of burn treatment. Which snack would be best for the nurse to offer to this patient? a. Bananas c. Vanilla milkshake b. Orange gelatin d. Whole grain bagel

ANS: C A patient with a burn injury needs high-protein and high-calorie food intake, and the milkshake is the highest in these nutrients. The other choices are not as nutrient dense as the milkshake. Gelatin is likely high in sugar. The bagel is a good carbohydrate choice but low in protein. Bananas are a good source of potassium but are not high in protein and calories. DIF: Cognitive Level: Analyze (analysis)

The emergency department (ED) nurse receives report that a patient involved in a motor vehicle crash is being transported to the facility with an estimated arrival in 1 minute. In preparation for the patient's arrival, the nurse will obtain a. hypothermia blanket. b. lactated Ringer's solution. c. two 14-gauge IV catheters. d. dopamine (Intropin) infusion.

ANS: C A patient with multiple trauma may require fluid resuscitation to prevent or treat hypovolemic shock, so the nurse will anticipate the need for 2 large bore IV lines to administer normal saline. Lactated Ringer's solution should be used cautiously and will not be ordered until the patient has been assessed for possible liver abnormalities. Vasopressor infusion is not used as the initial therapy for hypovolemic shock. Patients in shock need to be kept warm not cool.

A patient has just arrived in the emergency department after an electrical burn from exposure to a high-voltage current. What is the priority nursing assessment? a. Oral temperature c. Extremity movement b. Peripheral pulses d. Pupil reaction to light

ANS: C All patients with electrical burns should be considered at risk for cervical spine injury, and assessment of extremity movement will provide baseline data. The other assessment data are also necessary but not as essential as determining the cervical spine status. DIF: Cognitive Level: Analyze (analysis)

Which patient is most appropriate for the burn unit charge nurse to assign to a registered nurse (RN) who has floated from the hospital medical unit? a. A patient who has twice-daily burn debridements to partial-thickness facial burns b. A patient who has just returned from having a cultured epithelial autograft to the chest c. A patient who has a weight loss of 15% from admission and will have enteral feedings started d. A patient who has blebs under an autograft on the thigh and has an order for bleb aspiration

ANS: C An RN from a medical unit would be familiar with malnutrition and with administration and evaluation of response to enteral feedings. The other patients require burn assessment and care that is more appropriate for staff who regularly care for burned patients. DIF: Cognitive Level: Analyze (analysis)

A 22-yr-old patient who experienced a drowning accident in a local pool, but now is awake and breathing spontaneously, is admitted for observation. Which assessment will be most important for the nurse to take during the observation period? a. Auscultate heart sounds. c. Auscultate breath sounds. b. Palpate peripheral pulses. d. Check mental orientation.

ANS: C Because pulmonary edema is a common complication after drowning, the nurse should assess the breath sounds frequently. The other information also will be obtained by the nurse, but it is not as pertinent to the patient's admission diagnosis. DIF: Cognitive Level: Analyze (analysis)

Which finding about a patient who is receiving vasopressin (Pitressin) to treat septic shock is most important for the nurse to communicate to the health care provider? a. The patient's urine output is 18 mL/hr. b. The patient's heart rate is 110 beats/minute. c. The patient is complaining of chest pain. d. The patient's peripheral pulses are weak.

ANS: C Because vasopressin is a potent vasoconstrictor, it may decrease coronary artery perfusion. The other information is consistent with the patient's diagnosis and should be reported to the health care provider but does not indicate a need for a change in therapy.

Which nursing action is a priority for a patient who has suffered a burn injury while working on an electrical power line? a. Inspect the contact burns. c. Stabilize the cervical spine. b. Check the blood pressure. d. Assess alertness and orientation.

ANS: C Cervical spine injuries are commonly associated with electrical burns. Therefore stabilization of the cervical spine takes precedence after airway management. The other actions are also included in the emergent care after electrical burns, but the most important action is to avoid spinal cord injury. DIF: Cognitive Level: Analyze (analysis)

Esomeprazole (Nexium) is prescribed for a patient who incurred extensive burn injuries 5 days ago. Which nursing assessment would best evaluate the effectiveness of the drug? a. Bowel sounds c. Stool occult blood b. Stool frequency d. Abdominal distention

ANS: C H2 blockers and proton pump inhibitors are given to prevent Curling's ulcer in the patient who has sustained burn injuries. Proton pump inhibitors usually do not affect bowel sounds, stool frequency, or appetite. DIF: Cognitive Level: Apply (application)

A patient with severe burns has crystalloid fluid replacement ordered using the Parkland formula. The initial volume of fluid to be administered in the first 24 hours is 30,000 mL. The initial rate of administration is 1875 mL/hr. After the first 8 hours, what rate should the nurse infuse the IV fluids? a. 219 mL/hr c. 938 mL/hr b. 625 mL/hr d. 1875 mL/hr

ANS: C Half of the fluid replacement using the Parkland formula is administered in the first 8 hours and the other half over the next 16 hours. In this case, the patient should receive half of the initial rate, or 938 mL/hr. DIF: Cognitive Level: Apply (application)

The nurse is reviewing laboratory results on a patient who had a large burn 48 hours ago. Which result requires priority action by the nurse? a. Hematocrit of 53% c. Serum potassium of 6.1 mEq/L b. Serum sodium of 147 mEq/L d. Blood urea nitrogen of 37 mg/dL

ANS: C Hyperkalemia can lead to life-threatening dysrhythmias and indicates that the patient requires cardiac monitoring and immediate treatment to lower the potassium level. The other laboratory values are also abnormal and require changes in treatment, but they are not as immediately life threatening as the elevated potassium level. DIF: Cognitive Level: Analyze (analysis)

A 19-year-old patient with massive trauma and possible spinal cord injury is admitted to the emergency department (ED). Which assessment finding by the nurse will help confirm a diagnosis of neurogenic shock? a. Inspiratory crackles. b. Cool, clammy extremities. c. Apical heart rate 45 beats/min. d. Temperature 101.2° F (38.4° C).

ANS: C Neurogenic shock is characterized by hypotension and bradycardia. The other findings would be more consistent with other types of shock

An older patient with cardiogenic shock is cool and clammy and hemodynamic monitoring indicates a high systemic vascular resistance (SVR). Which intervention should the nurse anticipate doing next? a. Increase the rate for the dopamine (Intropin) infusion. b. Decrease the rate for the nitroglycerin (Tridil) infusion. c. Increase the rate for the sodium nitroprusside (Nipride) infusion. d. Decrease the rate for the 5% dextrose in normal saline (D5/.9 NS) infusion.

ANS: C Nitroprusside is an arterial vasodilator and will decrease the SVR and afterload, which will improve cardiac output. Changes in the D5/.9 NS and nitroglycerin infusions will not directly decrease SVR. Increasing the dopamine will tend to increase SVR.

The patient with neurogenic shock is receiving a phenylephrine (Neo-Synephrine) infusion through a right forearm IV. Which assessment finding obtained by the nurse indicates a need for immediate action? a. The patient's heart rate is 58 beats/minute. b. The patient's extremities are warm and dry. c. The patient's IV infusion site is cool and pale. d. The patient's urine output is 28 mL over the last hour.

ANS: C The coldness and pallor at the infusion site suggest extravasation of the phenylephrine. The nurse should discontinue the IV and, if possible, infuse the medication into a central line. An apical pulse of 58 is typical for neurogenic shock but does not indicate an immediate need for nursing intervention. A 28-mL urinary output over 1 hour would require the nurse to monitor the output over the next hour, but an immediate change in therapy is not indicated. Warm, dry skin is consistent with early neurogenic shock, but it does not indicate a need for a change in therapy or immediate action.

A patient who has been involved in a motor vehicle crash arrives in the emergency department (ED) with cool, clammy skin; tachycardia; and hypotension. Which intervention ordered by the health care provider should the nurse implement first? a. Insert two large-bore IV catheters. b. Initiate continuous electrocardiogram (ECG) monitoring. c. Provide oxygen at 100% per non-rebreather mask. d. Draw blood to type and crossmatch for transfusions.

ANS: C The first priority in the initial management of shock is maintenance of the airway and ventilation. ECG monitoring, insertion of IV catheters, and obtaining blood for transfusions should also be rapidly accomplished but only after actions to maximize oxygen delivery have been implemented.

On admission to the burn unit, a patient with an approximate 25% total body surface area (TBSA) burn has the following initial laboratory results: Hct 58%, Hgb 18.2 mg/dL (172 g/L), serum K+ 4.9 mEq/L (4.8 mmol/L), and serum Na+ 135 mEq/L (135 mmol/L). Which of the following prescribed actions should be the nurse's priority? a. Monitoring urine output every 4 hours. b. Continuing to monitor the laboratory results. c. Increasing the rate of the ordered IV solution. d. Typing and crossmatching for a blood transfusion.

ANS: C The patient's laboratory results show hemoconcentration, which may lead to a decrease in blood flow to the microcirculation unless fluid intake is increased. Because the hematocrit and hemoglobin are elevated, a transfusion is inappropriate, although transfusions may be needed after the emergent phase once the patient's fluid balance has been restored. On admission to a burn unit, the urine output would be monitored more often than every 4 hours (likely every1 hour). DIF: Cognitive Level: Analyze (analysis

A young adult patient who is in the rehabilitation phase after having deep partial-thickness face and neck burns has a nursing diagnosis of disturbed body image. Which statement by the patient best indicates that the problem is resolving? a. "I'm glad the scars are only temporary." b. "I will avoid using a pillow, so my neck will be OK." c. "Do you think dark beige makeup will cover this scar?" d. "I don't think my boyfriend will want to look at me now."

ANS: C The willingness to use strategies to enhance appearance is an indication that the disturbed body image is resolving. Expressing feelings about the scars indicates a willingness to discuss appearance but not resolution of the problem. Because deep partial-thickness burns leave permanent scars, a statement that the scars are temporary indicates denial rather than resolution of the problem. Avoiding using a pillow will help prevent contractures, but it does not address the problem of disturbed body image. DIF: Cognitive Level: Apply (application)

The emergency department (ED) nurse is starting therapeutic hypothermia in a patient who has been resuscitated after a cardiac arrest. Which actions in the hypothermia protocol can be delegated to an experienced licensed practical/vocational nurse (LPN/LVN) (select all that apply)? a. Continuously monitor heart rhythm. b. Assess neurologic status every 2 hours. c. Give acetaminophen (Tylenol) 650 mg. d. Place cooling blankets above and below patient. e. Attach rectal temperature probe to cooling blanket control panel.

ANS: C, D, E Experienced LPN/LVNs have the education and scope of practice to implement hypothermia measures (e.g., cooling blanket, temperature probe) and administer medications under the supervision of a registered nurse (RN). Assessment of neurologic status and monitoring the heart rhythm require RN-level education and scope of practice and should be done by the RN. DIF: Cognitive Level: Apply (application)

A patient who has burns on the arms, legs, and chest from a house fire has become agitated and restless 8 hours after being admitted to the hospital. Which action should the nurse take first? a. Stay at the bedside and reassure the patient. b. Administer the ordered morphine sulfate IV. c. Assess orientation and level of consciousness. d. Use pulse oximetry to check oxygen saturation.

ANS: D Agitation in a patient who may have suffered inhalation injury might indicate hypoxia, and this should be assessed by the nurse first. Administration of morphine may be indicated if the nurse determines that the agitation is caused by pain. Assessing level of consciousness and orientation is also appropriate but not as essential as determining whether the patient is hypoxemic. Reassurance is not helpful to reduce agitation in a hypoxemic patient. DIF: Cognitive Level: Analyze (analysis)

Which action will the nurse include in the plan of care for a patient in the rehabilitation phase after a burn injury to the right arm and chest? a. Keep the right arm in a position of comfort. b. Avoid the use of sustained-release narcotics. c. Teach about the purpose of tetanus immunization. d. Apply water-based cream to burned areas frequently.

ANS: D Application of water-based emollients will moisturize new skin and decrease flakiness and itching. To avoid contractures, the joints of the right arm should be positioned in an extended position, which is not the position of comfort. Patients may need to continue the use of opioids during rehabilitation. Tetanus immunization would have been given during the emergent phase of the burn injury. DIF: Cognitive Level: Apply (application)

Which assessment information is most important for the nurse to obtain to evaluate whether treatment of a patient with anaphylactic shock has been effective? a. Heart rate b. Orientation c. Blood pressure d. Oxygen saturation

ANS: D Because the airway edema that is associated with anaphylaxis can affect airway and breathing, the oxygen saturation is the most critical assessment. Improvements in the other assessments will also be expected with effective treatment of anaphylactic shock.

5. A 19-yr-old patient is brought to the emergency department (ED) with multiple lacerations and tissue avulsion of the left hand. When asked about tetanus immunization, the patient denies having any previous vaccinations. The nurse will anticipate giving a. tetanus immunoglobulin (TIG) only. b. TIG and tetanus-diphtheria toxoid (Td). c. tetanus-diphtheria toxoid and pertussis vaccine (Tdap) only. d. TIG and tetanus-diphtheria toxoid and pertussis vaccine (Tdap).

ANS: D For an adult with no previous tetanus immunizations, TIG and Tdap are recommended. The other immunizations are not sufficient for this patient.

The emergency department (ED) triage nurse is assessing four victims involved in a motor vehicle collision. Which patient has the highest priority for treatment? a. A patient with no pedal pulses b. A patient with an open femur fracture c. A patient with bleeding facial lacerations d. A patient with paradoxical chest movement

ANS: D Most immediate deaths from trauma occur because of problems with ventilation, so the patient with paradoxical chest movements should be treated first. Face and head fractures can obstruct the airway, but the patient with facial injuries only has lacerations. The other two patients also need rapid intervention but do not have airway or breathing problems. DIF: Cognitive Level: Analyze (analysis)

Which prescribed drug is best for the nurse to give before scheduled wound debridement on a patient with partial-thickness burns? a. ketorolac c. gabapentin (Neurontin) b. lorazepam (Ativan) d. hydromorphone (Dilaudid)

ANS: D Opioid pain medications are the best choice for pain control. The other drugs are used as adjuvants to enhance the effects of opioids. DIF: Cognitive Level: Analyze (analysis)

When the nurse educator is evaluating the skills of a new registered nurse (RN) caring for patients experiencing shock, which action by the new RN indicates a need for more education? a. Placing the pulse oximeter on the ear for a patient with septic shock b. Keeping the head of the bed flat for a patient with hypovolemic shock c. Increasing the nitroprusside (Nipride) infusion rate for a patient with a high SVR d. Maintaining the room temperature at 66° to 68° F for a patient with neurogenic shock

ANS: D Patients with neurogenic shock may have poikilothermia. The room temperature should be kept warm to avoid hypothermia. The other actions by the new RN are appropriate.

To evaluate the effectiveness of the pantoprazole (Protonix) ordered for a patient with systemic inflammatory response syndrome (SIRS), which assessment will the nurse perform? a. Auscultate bowel sounds. b. Palpate for abdominal pain. c. Ask the patient about nausea. d. Check stools for occult blood.

ANS: D Proton pump inhibitors are given to decrease the risk for stress ulcers in critically ill patients. The other assessments also will be done, but these will not help in determining the effectiveness of the pantoprazole administration.

During the emergent phase of burn care, which assessment will be most useful in determining whether the patient is receiving adequate fluid infusion? a. Check skin turgor. c. Assess mucous membranes. b. Monitor daily weight. d. Measure hourly urine output.

ANS: D When fluid intake is adequate, the urine output will be at least 0.5 to 1 mL/kg/hr. The patient's weight is not useful in this situation because of the effects of third spacing and evaporative fluid loss. Mucous membrane assessment and skin turgor also may be used, but they are not as adequate in determining that fluid infusions are maintaining adequate perfusion. DIF: Cognitive Level: Analyze (analysis)

........

........

The nurse is caring for a patient admitted with a drug overdose. What is the nurses priority responsibility in caring for this patient? A) Support the patients respiratory and cardiovascular function. B) Provide for the safety of the patient. C) Enhance clearance of the offending agent. D) Ensure the safety of the staff.

A Feedback: Treatment goals for a patient with a drug overdose are to support the respiratory and cardiovascular functions, to enhance clearance of the agent, and to provide for safety of the patient and staff. Of these responsibilities, however, support of vital physiologic function is a priority.

The nurse is providing care for an older adult patient who is experiencing low partial pressure of oxygen in arterial blood (PaO2) as a result of worsening left-sided pneumonia. Which intervention should the nurse use to help the patient mobilize his secretions? A Augmented coughing or huff coughing B Positioning the patient side-lying on his left side C Frequent and aggressive nasopharyngeal suctioning D Application of noninvasive positive pressure ventilation (NIPPV)

A Augmented coughing or huff coughing (Augmented coughing and huff coughing techniques may aid the patient in the mobilization of secretions. If positioned side-lying, the patient should be positioned on his right side (good lung down) for improved perfusion and ventilation. Suctioning may be indicated but should always be performed cautiously because of the risk of hypoxia. NIPPV is inappropriate in the treatment of patients with excessive secretions.)

2. The nurse is caring for a patient who has an intraaortic balloon pump (IABP) after a massive heart attack. When assessing the patient, the nurse notices blood backing up into the IABP catheter. In which order should the nurse take the following actions? (Put a comma and a space between each answer choice [A, B, C, D].) a. Confirm that the IABP console has turned off. b. Assess the patient's vital signs and orientation. c. Obtain supplies for insertion of a new IABP catheter. d. Notify the health care provider of the IABP malfunction.

A, B, D, C

A client recovering from an open reduction of the femur suddenly feels light-headed, with increased anxiety and agitation. Which key vital sign differentiates a pulmonary embolism from early sepsis? A. Temperature B. Pulse C. Respiration D. Blood pressure

A. Temperature A sign of early sepsis is low-grade fever. Both early sepsis and thrombus may cause tachycardia, tachypnea, and hypotension.

An unresponsive patient is admitted to the emergency department (ED) after falling through the ice while ice skating. Which assessment will the nurse obtain first? a. Pulse b. Heart rhythm c. Breath sounds d. Body temperature

ANS: A The priority assessment in an unresponsive patient relates to CAB (circulation, airway, breathing) so a pulse check should be performed first. While assessing the pulse, the nurse should look for signs of breathing. The other data will also be collected rapidly but are not as essential as determining if there is a pulse. DIF: Cognitive Level: Apply (application)

A nurse is assessing a patient who is receiving a nitroprusside (Nipride) infusion to treat cardiogenic shock. Which finding indicates that the medication is effective? a. No new heart murmurs b. Decreased troponin level c. Warm, pink, and dry skin d. Blood pressure 92/40 mm Hg

ANS: C Warm, pink, and dry skin indicates that perfusion to tissues is improved. Since nitroprusside is a vasodilator, the blood pressure may be low even if the medication is effective. Absence of a heart murmur and a decrease in troponin level are not indicators of improvement in shock.

A patient who was found unconscious in a burning house is brought to the emergency department by ambulance. The nurse notes that the patient's skin color is bright red. Which action should the nurse take first? a. Insert two large-bore IV lines. b. Check the patient's orientation. c. Assess for singed nasal hair and dark oral mucous membranes. d. Place the patient on 100% O2using a nonrebreather mask.

ANS: D The patient's history and skin color suggest carbon monoxide poisoning, which should be treated by rapidly starting O2 at 100%. The other actions can be taken after the action to correct gas exchange. DIF: Cognitive Level: Analyze (analysis)

1. A patient who has been in the intensive care unit for 4 days has disturbed sensory perception from sleep deprivation. Which action should the nurse include in the plan of care? a. Administer prescribed sedatives or opioids at bedtime to promote sleep. b. Cluster nursing activities so that the patient has uninterrupted rest periods. c. Silence the alarms on the cardiac monitors to allow 30- to 40-minute naps. d. Eliminate assessments between 2200 and 0600 to allow uninterrupted sleep.

B

12. The central venous oxygen saturation (ScvO2) is decreasing in a patient who has severe pancreatitis. To determine the possible cause of the decreased ScvO2, the nurse assesses the patient's a. lipase level. c. urinary output. b. temperature. d. body mass index.

B

14. The nurse is caring for a patient who has an intraaortic balloon pump in place. Which action should be included in the plan of care? a. Avoid the use of anticoagulant medications. b. Measure the patient's urinary output every hour. c. Provide passive range of motion for all extremities. d. Position the patient supine with head flat at all times.

B

16. To verify the correct placement of an oral endotracheal tube (ET) after insertion, the best initial action by the nurse is to a. obtain a portable chest x-ray. b. use an end-tidal CO2 monitor. c. auscultate for bilateral breath sounds. d. observe for symmetrical chest movement.

B

2. Which hemodynamic parameter best reflects the effectiveness of drugs that the nurse gives to reduce a patient's left ventricular afterload? a. Mean arterial pressure (MAP) b. Systemic vascular resistance (SVR) c. Pulmonary vascular resistance (PVR) d. Pulmonary artery wedge pressure (PAWP)

B

26. An 81-yr-old patient who has been in the intensive care unit (ICU) for a week is now stable and transfer to the progressive care unit is planned. On rounds, the nurse notices that the patient has new onset confusion. The nurse will plan to a. give PRN lorazepam (Ativan) and cancel the transfer. b. inform the receiving nurse and then transfer the patient. c. notify the health care provider and postpone the transfer. d. obtain an order for restraints as needed and transfer the patient.

B

28. The nurse is caring for a patient who has an arterial catheter in the left radial artery for arterial pressure-based cardiac output (APCO) monitoring. Which information obtained by the nurse requires a report to the health care provider? a. The patient has a positive Allen test result. b. There is redness at the catheter insertion site. c. The mean arterial pressure (MAP) is 86 mm Hg. d. The dicrotic notch is visible in the arterial waveform.

B

3. While close family members are visiting, a patient has a respiratory arrest, and resuscitation is started. Which action by the nurse is best? a. Tell the family members that watching the resuscitation will be very stressful. b. Ask family members if they wish to remain in the room during the resuscitation. c. Take the family members quickly out of the patient room and remain with them. d. Assign a staff member to wait with family members just outside the patient room.

B

33. The nurse educator is evaluating the performance of a new registered nurse (RN) who is providing care to a patient who is receiving mechanical ventilation with 15 cm H2O of peak end-expiratory pressure (PEEP). Which action indicates that the new RN is safe? a. The RN plans to suction the patient every 1 to 2 hours. b. The RN uses a closed-suction technique to suction the patient. c. The RN tapes the connection between the ventilator tubing and the ET. d. The RN changes the ventilator circuit tubing routinely every 48 hours.

B

4. After surgery for an abdominal aortic aneurysm, a patient's central venous pressure (CVP) monitor indicates low pressures. Which action should the nurse take? a. Administer IV diuretic medications. b. Increase the IV fluid infusion per protocol. c. Increase the infusion rate of IV vasodilators. d. Elevate the head of the patient's bed to 45 degrees.

B

6. The intensive care unit (ICU) nurse educator determines that teaching a new staff nurse about arterial pressure monitoring has been effective when the nurse a. balances and calibrates the monitoring equipment every 2 hours. b. positions the zero-reference stopcock line level with the phlebostatic axis. c. ensures that the patient is supine with the head of the bed flat for all readings. d. rechecks the location of the phlebostatic axis with changes in the patient's position.

B

A male patient with multiple injuries is brought to the ED by ambulance. He has had his airway stabilized and is breathing on his own. The ED nurse does not see any active bleeding, but should suspect internal hemorrhage based on what finding? A) Absence of bruising at contusion sites B) Rapid pulse and decreased capillary refill C) Increased BP with narrowed pulse pressure D) Sudden diaphoresis

B Feedback: The nurse would anticipate that the pulse would increase and BP would decrease. Urine output would also decrease. An absence of bruising and the presence of diaphoresis would not suggest internal hemorrhage.

A patient is brought to the hospital in cardiac arrest by emergency personnel who are performing resuscitation. The spouse arrives as the patient is taken into a treatment room and asks to stay with the patient. The nurse should a. have the spouse wait outside the treatment room with a designated staff member to provide emotional support. b. bring the spouse into the room and ensure him or her that a member of the team will explain the care given and answer questions. c. explain that the presence of family members is distracting to staff and might impair the resuscitation efforts. d. advise the spouse that if the resuscitation effort is unsuccessful, the memories may have an adverse impact on grieving.

B Rationale: Family members and patients report benefits from family presence during resuscitation efforts, so the nurse should try to accommodate the spouse. Having the spouse wait outside the room is not as supportive to the spouse or patient. It would be inappropriate to imply that the spouse's presence would have adverse consequences for the patient. Family members do not report problems with grieving caused by being present during resuscitation efforts.

A patient has experienced blunt abdominal trauma from a motor vehicle accident. The nurse should explain to the patient the purpose of a. magnetic resonance imaging (MRI). b. ultrasonography. c. peritoneal lavage. d. nasogastric (NG) tube placement.

B Rationale: If intra-abdominal bleeding is suspected, focused abdominal ultrasonography is obtained to look for intraperitoneal bleeding. MRI would not be used. Peritoneal lavage is an alternative, but it is more invasive. An NG tube would not be helpful in diagnosis of intra-abdominal bleeding.

A patient has been brought to the emergency department with a gunshot wound to the abdomen. In obtaining a history of the incident to determine possible injuries, the nurse asks a. "Where did the incident occur?" b. "What direction did the bullet enter the body?" c. "How long ago did the incident happen?" d. "What emergency care was started at the scene?"

B Rationale: The entry point and direction of the bullet will help to predict the type of injuries the patient has. The other information is not as useful in determining which diagnostic studies and care are needed immediately.

When caring for a patient with head and neck trauma after a motorcycle accident, the emergency department nurse's first action should be to a. suction the mouth and oropharynx. b. immobilize the cervical spine. c. administer supplemental oxygen. d. obtain venous access.

B Rationale: When there is a risk of spinal cord injury, the nurse's initial action is immobilization of the cervical spine during positioning of the head and neck for airway management. Suctioning, supplemental oxygen administration, and venous access are also necessary after the cervical spine is protected by immobilization.

A 13-year-old is being admitted to the ED after falling from a roof and sustaining blunt abdominal injuries. To assess for internal injury in the patients peritoneum, the nurse should anticipate what diagnostic test? A) Radiograph B) Computed tomography (CT) scan C) Complete blood count (CBC) D) Barium swallow

B Feedback: CT scan of the abdomen, diagnostic peritoneal lavage, and abdominal ultrasound are appropriate diagnostic tools to assess intra-abdominal injuries. X-rays do not yield sufficient data and a CBC would not reveal the presence of intraperitoneal injury.

A trauma client with multiple open wounds is brought to the emergency department in cardiac arrest. Which action should the nurse take prior to providing advanced cardiac life support? a. Contact the on-call orthopedic surgeon. b. Don personal protective equipment. c. Notify the Rapid Response Team. d. Obtain a complete history from the paramedic.

B Nurses must recognize and plan for a high risk of contamination with blood and body fluids when engaging in trauma resuscitation. Standard Precautions should be taken in all resuscitation situations and at other times when exposure to blood and body fluids is likely. Proper attire consists of an impervious cover gown, gloves, eye protection, a facemask, a surgical cap, and shoe covers.

The emergency department team is performing cardiopulmonary resuscitation on a client when the client's spouse arrives at the emergency department. Which action should the nurse take first? a. Request that the client's spouse sit in the waiting room. b. Ask the spouse if he wishes to be present during the resuscitation. c. Suggest that the spouse begin to pray for the client. d. Refer the client's spouse to the hospitals crisis team.

B If resuscitation efforts are still under way when the family arrives, one or two family members may be given the opportunity to be present during lifesaving procedures. The other options do not give the spouse the opportunity to be present for the client or to begin to have closure.

A nurse is evaluating levels and functions of trauma centers. Which function is appropriately paired with the level of the trauma center? a. Level I Located within remote areas and provides advanced life support within resource capabilities b. Level II Located within community hospitals and provides care to most injured clients c. Level III Located in rural communities and provides only basic care to clients d. Level IV Located in large teaching hospitals and provides a full continuum of trauma care for all clients

B Level I trauma centers are usually located in large teaching hospital systems and provide a full continuum of trauma care for all clients. Both Level II and Level III facilities are usually located in community hospitals. These trauma centers provide care for most clients and transport to Level I centers when client needs exceed resource capabilities. Level IV trauma centers are usually located in rural and remote areas. These centers provide basic care, stabilization, and advanced life support while transfer arrangements to higher-level trauma centers are made.

When caring for older adult patients with respiratory failure, the nurse will add which intervention to individualize care? A Position the patient in the supine position primarily. B Assess frequently for signs and symptoms of delirium. C Provide early endotracheal intubation to reduce complications. D Delay activity and ambulation to provide additional healing time.

B Assess frequently for signs and symptoms of delirium. (Older adult patients are more predisposed to factors such as delirium, health care associated infections, and polypharmacy. Individualizing the older patient's care plan to address these factors will improve care. Older adult patients are not required to remain in a supine position only and should increase activity as soon as stability is determined. Endotracheal intubation is not provided early, and noninvasive positive pressure ventilation may be considered as an alternative. The nurse should consider that the aging process leads to decreased lung elastic recoil, weakened lung muscles and reduced gas exchange, which may make the patient difficult to wean from the ventilator.)

When caring for a patient with acute respiratory distress syndrome (ARDS), which finding indicates therapy is appropriate? A pH is 7.32. B PaO2 is greater than or equal to 60 mm Hg. C PEEP increased to 20 cm H2O caused BP to fall to 80/40. D No change in PaO2 when patient is turned from supine to prone position

B PaO2 is greater than or equal to 60 mm Hg. (The overall goal in caring for the patient with ARDS is for the PaO2 to be greater than or equal to 60 mm Hg with adequate lung ventilation to maintain a normal pH of 7.35 to 7.45. PEEP is usually increased for ARDS patients, but a dramatic reduction in BP indicates a complication of decreased cardiac output. A positive occurrence is a marked improvement in PaO2 from perfusion better matching ventilation when the anterior air-filled, nonatelectatic alveoli become dependent in the prone position.)

A nurse is triaging clients in the emergency department (ED). Which client should the nurse prioritize to receive care first? a. A 22-year-old with a painful and swollen right wrist b. A 45-year-old reporting chest pain and diaphoresis c. A 60-year-old reporting difficulty swallowing and nausea d. An 81-year-old with a respiratory rate of 28 breaths/min and a temperature of 101 F

B ~ A client experiencing chest pain and diaphoresis would be classified as emergent and would be triaged immediately to a treatment room in the ED. The other clients are more stable.

A nurse is caring for clients in a busy emergency department. Which actions should the nurse take to ensure client and staff safety? (SATA) a. Leave the stretcher in the lowest position with rails down so that the client can access the bathroom. b. Use two identifiers before each intervention and before mediation administration. c. Attempt de-escalation strategies for clients who demonstrate aggressive behaviors. d. Search the belongings of clients with altered mental status to gain essential medical information. e. Isolate clients who have immune suppression disorders to prevent hospital-acquired infections.c

B, C, D To ensure client and staff safety, nurses should use two identifiers per The Joint Commissions National Patient Safety Goals; follow the hospitals security plan, including de-escalation strategies for people who demonstrate aggressive or violent tendencies; and search belongings to identify essential medical information. Nurses should also use standard fall prevention interventions, including leaving stretchers in the lowest position with rails up, and isolating clients who present with signs and symptoms of contagious infectious disorders.

5. The hemodynamic changes the nurse expects to find after successful initiation of intraaortic balloon pump therapy in a patient with cardiogenic shock include (select all that apply) a. decreased SV. b. decreased SVR. c. decreased PAWP. d. increased diastolic BP. e. decreased myocardial O2 consumption.

B, C, D, E

An emergency room nurse is caring for a trauma client. Which interventions should the nurse perform during the primary survey? (SATA) a. Foley catheterization b. Needle decompression c. Initiating IV fluids d. Splinting open fractures e. Endotracheal intubation f. Removing wet clothing g. Laceration repair

B, C, E, F The primary survey for a trauma client organizes the approach to the client so that life-threatening injuries are rapidly identified and managed. The primary survey is based on the standard mnemonic ABC, with an added D and E: Airway and cervical spine control; Breathing; Circulation; Disability; and Exposure. After the completion of primary diagnostic and laboratory studies, and the insertion of gastric and urinary tubes, the secondary survey (a complete head-to-toe assessment) can be carried out.

A client with septic shock has been started on dopamine (Intropin) at 12 mcg/kg/min. Which response indicates a positive outcome? A. Hourly urine output 10 to 12 mL/hr B. Blood pressure 90/60 mm Hg and mean arterial pressure 70 mm Hg C. Blood glucose 245 mg/dL D. Serum creatinine 3.6 mg/dL

B. Blood pressure 90/60 mm Hg and mean arterial pressure 70 mm Hg Dopamine improves blood flow by increasing peripheral resistance, which increases blood pressure—a positive response in this case. Urine output less than 30 mL/hr or 0.5 mL/kg/hr and elevations in serum creatinine indicate poor tissue perfusion to the kidney and are a negative consequence of shock, not a positive response. Although a blood glucose of 245 mg/dL is an abnormal finding, dopamine increases blood pressure and myocardial contractility, not glucose levels.

Which problem in the clients below best demonstrates the highest risk for hypovolemic shock? A. Client receiving a blood transfusion B. Client with severe ascites C. Client with myocardial infarction D. Client with syndrome of inappropriate antidiuretic hormone (SIADH) secretion

B. Client with severe ascites Fluid shifts from vascular to intra-abdominal may cause decreased circulating blood volume and poor tissue perfusion. Volume depletion is only one reason why a person may require a blood transfusion; anemia is another. The client receiving a blood transfusion does not have as high a risk as the client with severe ascites. Myocardial infarction results in tissue necrosis in the heart muscle; no blood or fluid losses occur. Owing to excess antidiuretic hormone secretion, the client with SIADH will retain fluid and therefore is not at risk for hypovolemic shock.

Which laboratory result is seen in late sepsis? A. Decreased serum lactate B. Decreased segmented neutrophil count C. Increased numbers of monocytes D. Increased platelet count

B. Decreased segmented neutrophil count A decreased segmented neutrophil count is indicative of late sepsis. Serum lactate is increased in late sepsis. Monocytosis is usually seen in diseases such as tuberculosis and Rocky Mountain spotted fever. An increased platelet count does not indicate sepsis; late in sepsis, platelets may decrease due to consumptive coagulopathy.

The nurse plans to administer an antibiotic to a client newly admitted with septic shock. What action does the nurse take first? A. Administer the antibiotic immediately. B. Ensure that blood cultures were drawn. C. Obtain signature for informed consent. D. Take the client's vital signs.

B. Ensure that blood cultures were drawn. Cultures must be taken to identify the organism for more targeted antibiotic treatment before antibiotics are administered. Antibiotics are not administered until after all cultures are taken. A signed consent is not needed for medication administration. Monitoring the client's vital signs is important, but the antibiotic must be administered within 1 to 3 hours; timing is essential.

How does the nurse caring for a client with septic shock recognize that severe tissue hypoxia is present? A. PaCO2 58 mm Hg B. Lactate 9.0 mmol/L C. Partial thromboplastin time 64 seconds D. Potassium 2.8 mEq/L

B. Lactate 9.0 mmol/L Poor tissue oxygenation at the cellular level causes anaerobic metabolism, with the by-product of lactic acid. Elevated partial pressure of carbon dioxide occurs with hypoventilation, which may be related to respiratory muscle fatigue, secretions, and causes other than hypoxia. Coagulation times reflect the ability of the blood to clot, not oxygenation at the cellular level. Elevation in potassium appears in septic shock due to acidosis; this value is decreased and is not consistent with septic shock.

Which clinical symptoms in a postoperative client indicate early sepsis with an excellent recovery rate if treated? A. Localized erythema and edema B. Low-grade fever and mild hypotension C. Low oxygen saturation rate and decreased cognition D. Reduced urinary output and increased respiratory rate

B. Low-grade fever and mild hypotension Low-grade fever and mild hypotension indicate very early sepsis, but with treatment, the probability of recovery is high. Localized erythema and edema indicate local infection. A low oxygen saturation rate and decreased cognition indicate active (not early) sepsis. Reduced urinary output and increased respiratory rate indicate severe sepsis.

A postoperative client is admitted to the intensive care unit with hypovolemic shock. Which nursing action does the nurse delegate to an experienced nursing assistant? A. Obtain vital signs every 15 minutes. B. Measure hourly urine output. C. Check oxygen saturation. D. Assess level of alertness.

B. Measure hourly urine output. Monitoring hourly urine output is included in nursing assistant education and does not require special clinical judgment; the nurse evaluates the results. Obtaining vital signs, monitoring oxygen saturation, and assessing mental status in critically ill clients requires the clinical judgment of the critical care nurse because immediate intervention may be needed.

13. An intraaortic balloon pump (IABP) is being used for a patient who is in cardiogenic shock. Which assessment data indicate to the nurse that the goals of treatment with the IABP are being met? a. Urine output of 25 mL/hr b. Heart rate of 110 beats/minute c. Cardiac output (CO) of 5 L/min d. Stroke volume (SV) of 40 mL/beat

C

19. Which assessment finding obtained by the nurse when caring for a patient receiving mechanical ventilation indicates the need for suctioning? a. The patient was last suctioned 6 hours ago. b. The patient's oxygen saturation drops to 93%. c. The patient's respiratory rate is 32 breaths/min. d. The patient has occasional audible expiratory wheezes.

C

20. The nurse notes thick, white secretions in the endotracheal tube (ET) of a patient who is receiving mechanical ventilation. Which intervention will most directly treat this finding? a. Reposition the patient every 1 to 2 hours. b. Increase suctioning frequency to every hour. c. Add additional water to the patient's enteral feedings. d. Instill 5 mL of sterile saline into the ET before suctioning.

C

23. A nurse is weaning a 68-kg patient who has chronic obstructive pulmonary disease (COPD) from mechanical ventilation. Which patient assessment finding indicates that the weaning protocol should be stopped? a. The patient's heart rate is 97 beats/min. b. The patient's oxygen saturation is 93%. c. The patient respiratory rate is 32 breaths/min. d. The patient's spontaneous tidal volume is 450 mL.

C

24. The nurse is caring for a patient receiving a continuous norepinephrine IV infusion. Which patient assessment finding indicates that the infusion rate may need to be adjusted? a. Heart rate is slow at 58 beats/min. b. Mean arterial pressure (MAP) is 56 mm Hg. c. Systemic vascular resistance (SVR) is elevated. d. Pulmonary artery wedge pressure (PAWP) is low.

C

27. The family members of a patient who has been admitted to the intensive care unit (ICU) with multiple traumatic injuries have just arrived in the ICU waiting room. Which action should the nurse take first? a. Explain ICU visitation policies and encourage family visits. b. Escort the family from the waiting room to the patient's bedside. c. Describe the patient's injuries and the care that is being provided. d. Invite the family to participate in an interprofessional care conference.

C

3. The critical care nurse recognizes that an ideal plan for caregiver involvement includes a. a caregiver at the bedside at all times. b. allowing caregivers at the bedside at preset, brief intervals. c. an individually devised plan to involve caregivers with care and comfort measures. d. restriction of visiting in the ICU because the environment is overwhelming to caregivers.

C

30. The nurse notes that a patient's endotracheal tube (ET), which was at the 22-cm mark, is now at the 25-cm mark, and the patient is anxious and restless. Which action should the nurse take next? a. Check the O2 saturation. b. Offer reassurance to the patient. c. Listen to the patient's breath sounds. d. Notify the patient's health care provider.

C

36. After change-of-shift report on a ventilator weaning unit, which patient should the nurse assess first? a. Patient who failed a spontaneous breathing trial and has been placed in a rest mode on the ventilator b. Patient who is intubated and has continuous partial pressure end-tidal CO2 (PETCO2) monitoring c. Patient who was successfully weaned and extubated 4 hours ago and has no urine output for the last 6 hours d. Patient with a central venous O2 saturation (ScvO2) of 69% while on bilevel positive airway pressure (BiPAP)

C

4. To establish hemodynamic monitoring for a patient, the nurse zeros the a. cardiac output monitoring system to the level of the left ventricle. b. pressure monitoring system to the level of the catheter tip located in the patient. c. pressure monitoring system to the level of the atrium, identified as the phlebostatic axis. d. pressure monitoring system to the level of the atrium, identified as the midclavicular line.

C

5. When caring for a patient with pulmonary hypertension, which parameter will the nurse use to directly evaluate the effectiveness of the treatment? a. Central venous pressure (CVP) b. Systemic vascular resistance (SVR) c. Pulmonary vascular resistance (PVR) d. Pulmonary artery wedge pressure (PAWP)

C

7. The nursing management of a patient with an artificial airway includes a. maintaining ET tube cuff pressure at 30 cm H2O. b. routine suctioning of the tube at least every 2 hours. c. observing for cardiac dysrhythmias during suctioning. d. preventing tube dislodgment by limiting mouth care to lubrication of the lips.

C

8. The nurse monitors the patient with positive pressure mechanical ventilation for a. paralytic ileus because pressure on the abdominal contents affects bowel motility. b. diuresis and sodium depletion because of increased release of atrial natriuretic peptide. c. signs of cardiovascular insufficiency because pressure in the chest impedes venous return. d. respiratory acidosis in a patient with COPD because of alveolar hyperventilation and increased PaO2 levels.

C

8. Which action should the nurse take when the low pressure alarm sounds for a patient who has an arterial line in the left radial artery? a. Fast flush the arterial line. b. Check the left hand for pallor. c. Assess for cardiac dysrhythmias. d. Re-zero the monitoring equipment.

C

A patient with multiple trauma is brought to the ED by ambulance after a fall while rock climbing. What is a responsibility of the ED nurse in this patients care? A) Intubating the patient B) Notifying family members C) Ensuring IV access D) Delivering specimens to the laboratory

C

A nurse is triaging clients in the emergency department. Which client should the nurse classify as nonurgent? a. A 44-year-old with chest pain and diaphoresis b. A 50-year-old with chest trauma and absent breath sounds c. A 62-year-old with a simple fracture of the left arm d. A 79-year-old with a temperature of 104 F

C A client in a nonurgent category can tolerate waiting several hours for health care services without a significant risk of clinical deterioration. The client with a simple arm fracture and palpable radial pulses is currently stable, is not at significant risk of clinical deterioration, and would be considered nonurgent. The client with chest pain and diaphoresis and the client with chest trauma are emergent owing to the potential for clinical deterioration and would be seen immediately. The client with a high fever may be stable now but also has a risk of deterioration.

A patient is experiencing respiratory insufficiency and cannot maintain spontaneous respirations. The nurse suspects that the physician will perform which of the following actions? A) Insert an oropharyngeal airway. B) Perform the jaw thrust maneuver. C) Perform endotracheal intubation. D) Perform a cricothyroidotomy.

C Feedback: Endotracheal tubes are used in cases when the patient cannot be ventilated with an oropharyngeal airway, which is used in patients who are breathing spontaneously. The jaw thrust maneuver does not establish an airway and cricothyroidotomy would be performed as a last resort.

A 24-year-old is brought to the emergency department with multiple lacerations and tissue avulsion of the right hand after catching the hand in a produce conveyor belt. When asked about tetanus immunization, the patient says, "I've never had any vaccinations." The nurse will anticipate administration of tetanus a. immunoglobulin. b. and diphtheria toxoid. c. immunoglobulin, tetanus-diphtheria toxoid, and pertussis vaccine. d. immunoglobulin and tetanus-diphtheria toxoid.

C Rationale: For a patient with unknown immunization status, the tetanus immune globulin is administered along with the Tdap (since the patient has not had pertussis vaccine previously). The other immunizations are not sufficient for this patient.

Four victims of an automobile crash are brought by ambulance to the emergency department. The triage nurse determines that the victim who has the highest priority for treatment is the one with a. severe bleeding of facial and head lacerations. b. an open femur fracture with profuse bleeding. c. a sucking chest wound. d. absence of peripheral pulses.

C Rationale: Most immediate deaths from trauma occur because of problems with ventilation, so the patient with a sucking chest wound should be treated first. Face and head fractures can obstruct the airway, but the patient with facial injuries has lacerations only. The other two patients also need rapid intervention but do not have airway or breathing problems.

A patient is brought to the ED by friends. The friends tell the nurse that the patient was using cocaine at a party. On arrival to the ED the patient is in visible distress with an axillary temperature of 40.1C (104.2F). What would be the priority nursing action for this patient? A) Monitor cardiovascular effects. B) Administer antipyretics. C) Ensure airway and ventilation. D) Prevent seizure activity.

C Feedback: Although all of the listed actions may be necessary for this patients care, the priority is to establish a patent airway and adequate ventilation.

An emergency room nurse assesses a client who has been raped. With which health care team member should the nurse collaborate when planning this client's care? a. Emergency medicine physician b. Case manager c. Forensic nurse examiner d. Psychiatric crisis nurse

C All other members of the health care team listed may be used in the management of this client's care. However, the forensic nurse examiner is educated to obtain client histories and collect evidence dealing with the assault, and can offer the counseling and follow-up needed when dealing with the victim of an assault.

An emergency room nurse is triaging victims of a multi-casualty event. Which client should receive care first? a. A 30-year-old distraught mother holding her crying child b. A 65-year-old conscious male with a head laceration c. A 26-year-old male who has pale, cool, clammy skin d. A 48-year-old with a simple fracture of the lower leg

C The client with pale, cool, clammy skin is in shock and needs immediate medical attention. The mother does not have injuries and so would be the lowest priority. The other two people need medical attention soon, but not at the expense of a person in shock.

A nurse is triaging clients in the emergency department. Which client should be considered urgent? a. A 20-year-old female with a chest stab wound and tachycardia b. A 45-year-old homeless man with a skin rash and sore throat c. A 75-year-old female with a cough and a temperature of 102 F d. A 50-year-old male with new-onset confusion and slurred speech

C A client with a cough and a temperature of 102 F is urgent. This client is at risk for deterioration and needs to be seen quickly, but is not in an immediately life-threatening situation. The client with a chest stab wound and tachycardia and the client with new-onset confusion and slurred speech should be triaged as emergent. The client with a skin rash and a sore throat is not at risk for deterioration and would be triaged as nonurgent.

While triaging clients in a crowded emergency department, a nurse assesses a client who presents with symptoms of tuberculosis. Which action should the nurse take first? a. Apply oxygen via nasal cannula. b. Administer intravenous 0.9% saline solution. c. Transfer the client to a negative-pressure room. d. Obtain a sputum culture and sensitivity.

C A client with signs and symptoms of tuberculosis or other airborne pathogens should be placed in a negative-pressure room to prevent contamination of staff, clients & family members in the crowded emergency department.

An emergency department (ED) case manager is consulted for a client who is homeless. Which intervention should the case manager provide? a. Communicate client needs and restrictions to support staff. b. Prescribe low-cost antibiotics to treat community-acquired infection. c. Provide referrals to subsidized community-based health clinics. d. Offer counseling for substance abuse and mental health disorders.

C Case management interventions include facilitating referrals to primary care providers who are accepting new clients or to subsidized community-based health clinics for clients or families in need of routine services. The ED nurse is accountable for communicating pertinent staff considerations, client needs, and restrictions to support staff (e.g., physical limitations, isolation precautions) to ensure that ongoing client and staff safety issues are addressed. The ED physician prescribes medications and treatments. The psychiatric nurse team evaluates clients with emotional behaviors or mental illness and facilitates the follow-up treatment plan, including possible admission to an appropriate psychiatric facility.

The nurse is admitting a 45-yr-old patient with asthma in acute respiratory distress. The nurse auscultates the patient's lungs and notes cessation of the inspiratory wheezing. The patient has not yet received any medication. What should this finding suggest to the nurse? A Spontaneous resolution of the acute asthma attack B An acute development of bilateral pleural effusions C Airway constriction requiring immediate interventions D Overworked intercostal muscles resulting in poor air exchange

C Airway constriction requiring immediate interventions (When a patient in respiratory distress has inspiratory wheezing and then it ceases, it is an indication of airway obstruction. This finding requires emergency action to restore airway patency. Cessation of inspiratory wheezing does not indicate spontaneous resolution of the acute asthma attack, bilateral pleural effusion development, or overworked intercostal muscles in this asthmatic patient that is in acute respiratory distress.)

The nurse is caring for a 37-yr-old female patient with multiple musculoskeletal injuries who has developed acute respiratory distress syndrome (ARDS). Which intervention should the nurse initiate to prevent stress ulcers? A Observe stools for frank bleeding and occult blood. B Maintain head of the bed elevation at 30 to 45 degrees. C Begin enteral feedings as soon as bowel sounds are present. D Administer prescribed lorazepam (Ativan) to reduce anxiety.

C Begin enteral feedings as soon as bowel sounds are present. (Stress ulcers prevention includes early initiation of enteral nutrition to protect the gastrointestinal (GI) tract from mucosal damage. Antiulcer agents such as histamine (H2)-receptor antagonists, proton pump inhibitors, and mucosal protecting agents are also indicated to prevent stress ulcers. Monitoring for GI bleeding does not prevent stress ulcers. Ventilator-associated pneumonia related to aspiration is prevented by elevation of the head of bed to 30 to 45 degrees Stress ulcers are not caused by anxiety. Stress ulcers are related to GI ischemia from hypotension, shock, and acidosis.)

The nurse is caring for a 27-yr-old man with multiple fractured ribs from a motor vehicle crash. Which clinical manifestation, if experienced by the patient, is an early indication that the patient is developing respiratory failure? A Tachycardia and pursed lip breathing B Kussmaul respirations and hypotension C Frequent position changes and agitation D Cyanosis and increased capillary refill time

C Frequent position changes and agitation (A change in mental status is an early indication of respiratory failure. The brain is sensitive to variations in oxygenation, arterial carbon dioxide levels, and acid-base balance. Restlessness, confusion, agitation, and combative behavior suggest inadequate oxygen delivery to the brain.)

The nurse is caring for a patient who is admitted with a barbiturate overdose. The patient is comatose with a blood pressure of 90/60 mm Hg, apical pulse of 110 beats/min, and respiratory rate of 8 breaths/min. Based on the initial assessment findings, the nurse recognizes that the patient is at risk for which type of respiratory failure? A Hypoxemic respiratory failure related to shunting of blood B Hypoxemic respiratory failure related to diffusion limitation C Hypercapnic respiratory failure related to alveolar hypoventilation D Hypercapnic respiratory failure related to increased airway resistance

C Hypercapnic respiratory failure related to alveolar hypoventilation (The patient's respiratory rate is decreased as a result of barbiturate overdose, which caused respiratory depression. The patient is at risk for hypercapnic respiratory failure due to an obtunded airway causing decreased respiratory rate and thus decreased CO2 elimination. Barbiturate overdose does not lead to shunting of blood, diffusion limitations, or increased airway resistance.)

Arterial blood gas results are reported to the nurse for a 68-yr-old patient admitted with pneumonia: pH 7.31, PaCO2 49 mm Hg, HCO3 26 mEq/L, and PaO2 52 mm Hg. What order should the nurse complete first? A Administer albuterol inhaler prn. B Increase fluid intake to 2500 mL per 24 hours. C Initiate oxygen at 2 liters/minute by nasal cannula. D Perform chest physical therapy four times per day.

C Initiate oxygen at 2 liters/minute by nasal cannula. (The arterial blood gas results indicate the patient is in uncompensated respiratory acidosis with moderate hypoxemia. Oxygen therapy is indicated to correct hypoxemia secondary to V/Q mismatch. Supplemental oxygen should be initiated at 1 to 3 L/min by nasal cannula, or 24% to 32% by simple face mask or Venturi mask to improve the PaO2. Albuterol would be administered next if needed for bronchodilation. Hydration is indicated for thick secretions, and chest physical therapy is indicated for patients with 30 mL or more of sputum production per day.)

An emergency department nurse is caring for a client who is homeless. Which action should the nurse take to gain the clients trust? a. Speak in a quiet and monotone voice. b. Avoid eye contact with the client. c. Listen to the client's concerns and needs. d. Ask security to store the client's belongings.

C ~ To demonstrate behaviors that promote trust with homeless clients, the emergency room nurse should make eye contact (if culturally appropriate), speak calmly, avoid any prejudicial or stereotypical remarks, show genuine care and concern by listening, and follow through on promises. The nurse should also respect the client's belongings and personal space.

The nurse is caring for a client in the refractory stage of cardiogenic shock. Which intervention does the nurse consider? A. Admission to rehabilitation hospital for ambulatory retraining B. Collaboration with home care agency for return to home C. Discussion with family and provider regarding palliative care D. Enrollment in a cardiac transplantation program

C. Discussion with family and provider regarding palliative care In this irreversible phase, therapy is not effective in saving the client's life, even if the cause of shock is corrected and mean arterial pressure temporarily returns to normal. A discussion on palliative care should be considered. Rehabilitation or returning home is unlikely. The client with sustained tissue hypoxia is not a candidate for organ transplantation.

What typical sign/symptom indicates the early stage of septic shock? A. Pallor and cool skin B. Blood pressure 84/50 mm Hg C. Tachypnea and tachycardia D. Respiratory acidosis

C. Tachypnea and tachycardia Signs of systemic inflammatory response syndrome, which precedes sepsis, include rapid respiratory rate, leukocytosis, and tachycardia. In the early stage of septic shock, the client is usually warm and febrile. Hypotension does not develop until later in septic shock due to compensatory mechanisms. Respiratory alkalosis occurs early in shock because of an increased respiratory rate.

1. Certification in critical care nursing (CCRN) by the American Association of Critical-Care Nurses indicates that the nurse a. is an advanced practice nurse who cares for acutely and critically ill patients. b. may practice independently to provide symptom management for the critically ill. c. has earned a master's degree in the field of advanced acute and critical care nursing. d. has practiced in critical care and successfully completed a test of critical care knowledge.

D

10. While assisting with the placement of a pulmonary artery (PA) catheter, the nurse notes that the catheter is correctly placed when the balloon is inflated and the monitor shows a a. typical PA pressure waveform. b. tracing of the systemic arterial pressure. c. tracing of the systemic vascular resistance. d. typical PA wedge pressure (PAWP) tracing.

D

15. While waiting for heart transplantation, a patient with severe cardiomyopathy has a ventricular assist device (VAD) implanted. When planning care for this patient, the nurse should anticipate a. preparing the patient for a permanent VAD. b. administering immunosuppressive medications. c. teaching the patient the reason for complete bed rest. d. monitoring the surgical incision for signs of infection.

D

17. To maintain proper cuff pressure of an endotracheal tube (ET) when the patient is on mechanical ventilation, the nurse should a. inflate the cuff with a minimum of 10 mL of air. b. inflate the cuff until the pilot balloon is firm on palpation. c. inject air into the cuff until a manometer shows 15 mm Hg pressure. d. inject air into the cuff until a slight leak is heard only at peak inflation.

D

18. The nurse notes premature ventricular contractions (PVCs) while suctioning a patient's endotracheal tube. Which next action by the nurse is indicated? a. Plan to suction the patient more frequently. b. Decrease the suction pressure to 80 mm Hg. c. Give antidysrhythmic medications per protocol. d. Stop and ventilate the patient with 100% oxygen.

D

21. Four hours after mechanical ventilation is initiated, a patient's arterial blood gas (ABG) results include a pH of 7.51, PaO2 of 82 mm Hg, PaCO2 of 26 mm Hg, and HCO3- of 23 mEq/L (23 mmol/L). The nurse will anticipate the need to a. increase the FIO2. c. increase the respiratory rate. b. increase the tidal volume. d. decrease the respiratory rate.

D

29. The nurse responds to a ventilator alarm and finds the patient lying in bed gasping and holding the endotracheal tube (ET) in her hand. Which action should the nurse take next? a. Activate the rapid response team. b. Provide reassurance to the patient. c. Call the health care provider to reinsert the tube. d. Manually ventilate the patient with 100% oxygen.

D

31. The nurse educator is evaluating the care that a new registered nurse (RN) provides to a patient receiving mechanical ventilation. Which action by the new RN indicates the need for more education? a. The RN increases the FIO2 to 100% before suctioning. b. The RN secures a bite block in place using adhesive tape. c. The RN asks for assistance to resecure the endotracheal tube. d. The RN positions the patient with the head of bed at 10 degrees.

D

34. The nurse is caring for a patient with a subarachnoid hemorrhage who is intubated and placed on a mechanical ventilator with 10 cm H2O of peak end-expiratory pressure (PEEP). When monitoring the patient, the nurse will need to notify the health care provider immediately if the patient develops a. O2 saturation of 93%. b. green nasogastric tube drainage. c. respirations of 20 breaths/minute. d. increased jugular venous distention.

D

37. After change-of-shift report, which patient should the progressive care nurse assess first? a. Patient who was extubated this morning and has a temperature of 101.4°F (38.6°C) b. Patient with bilevel positive airway pressure (BiPAP) for obstructive sleep apnea and a respiratory rate of 16 c. Patient with arterial pressure monitoring who is 2 hours post-percutaneous coronary intervention and needs to void d. Patient who is receiving IV heparin for a venous thromboembolism and has a partial thromboplastin time (PTT) of 101 sec

D

7. When monitoring the effectiveness of treatment for a patient with a large anterior wall myocardial infarction, the most pertinent measurement for the nurse to obtain is a. central venous pressure (CVP). b. systemic vascular resistance (SVR). c. pulmonary vascular resistance (PVR). d. pulmonary artery wedge pressure (PAWP).

D

9. Which nursing action is needed when preparing to assist with the insertion of a pulmonary artery catheter? a. Determine if the cardiac troponin level is elevated. b. Auscultate heart sounds before and during insertion. c. Place the patient on NPO status before the procedure. d. Attach cardiac monitoring leads before the procedure.

D

A workplace explosion has left a 40-year-old man burned over 65% of his body. His burns are second and third-degree burns, but he is conscious. How would this person be triaged? A. Green B. Yellow C. Red D. Black

D Feedback: The purpose of triaging in a disaster is to do the greatest good for the greatest number of people. The patient would be triaged as black due to the unlikelihood of survival. Persons triaged as green, yellow, or red have a higher chance of recovery.

A patient who has been exposed to anthrax is being treated in the local hospital. The nurse should prioritize what health assessments? A. Integumentary assessment B. Assessment for signs of hemorrhage C. Neurologic assessment D. Assessment of respiratory status

D Feedback: The second stage of anthrax infection by inhalation includes severe respiratory distress, including stridor, cyanosis, hypoxia, diaphoresis, hypotension, and shock. The first stage includes flu-like symptoms. The second stage of infection by inhalation does not include headache, vomiting, or syncope.

A patient is brought to the ER in an unconscious state. The physician notes that the patient is in need of emergency surgery. No family members are present, and the patient does not have identification. What action by the nurse is most important regarding consent for treatment? A) Ask the social worker to come and sign the consent. B) Contact the police to obtain the patients identity. C) Obtain a court order to treat the patient. D) Clearly document LOC and health status on the patients chart.

D Feedback: When patients are unconscious and in critical condition, the condition and situation should be documented to administer treatment quickly and timely when no consent can be obtained by usual routes. A social worker is not asked to sign the consent. Finding the patients identity is not a priority. Obtaining a court order would take too long.

An emergency department nurse is caring for a client who has died from a suspected homicide. Which action should the nurse take? a. Remove all tubes and wires in preparation for the medical examiner. b. Limit the number of visitors to minimize the family's trauma. c. Consult the bereavement committee to follow up with the grieving family. d. Communicate the client's death to the family in a simple and concrete manner.

D When dealing with client's and families in crisis, communicate in a simple and concrete manner to minimize confusion. Tubes must remain in place for the medical examiner. Family should be allowed to view the body. Offering to call for additional family support during the crisis is suggested. The bereavement committee should be consulted, but this is not the priority at this time.

Which patient would most benefit from noninvasive positive pressure ventilation (NIPPV) to promote oxygenation? A A patient whose cardiac output and blood pressure are unstable B A patient whose respiratory failure is due to a head injury with loss of consciousness C A patient with a diagnosis of cystic fibrosis and who is currently producing copious secretions D A patient who is experiencing respiratory failure as a result of the progression of myasthenia gravis

D A patient who is experiencing respiratory failure as a result of the progression of myasthenia gravis (NIPPV such as continuous positive airway pressure (CPAP) is most effective in treating patients with respiratory failure resulting from chest wall and neuromuscular disease. It is not recommended in patients who are experiencing hemodynamic instability, decreased level of consciousness, or excessive secretions.)

A 56-yr-old man with acute respiratory distress syndrome (ARDS) is on positive pressure ventilation (PPV). The patient's cardiac index is 1.4 L/min and pulmonary artery wedge pressure is 8 mm Hg. What order by the physician is important for the nurse to question? A Initiate a dobutamine infusion at 3 mcg/kg/min. B Administer 1 unit of packed red blood cells over the next 2 hours. C Change the maintenance intravenous (IV) rate from 75 to 125 mL/hr. D Increase positive end-expiratory pressure (PEEP) from 10 to 15 cm H2O.

D Increase positive end-expiratory pressure (PEEP) from 10 to 15 cm H2O. (Patients on PPV and PEEP frequently experience decreased cardiac output (CO) and cardiac index (CI). High levels of PEEP increase intrathoracic pressure and cause decreased venous return which results in decreased CO. Interventions to improve CO include lowering the PEEP, administering crystalloid fluids or colloid solutions, and use of inotropic drugs (e.g., dobutamine, dopamine). Packed red blood cells may also be administered to improve CO and oxygenation if the hemoglobin is less than 9 or 10 mg/dL.)

A 72-yr-old woman with aspiration pneumonia develops severe respiratory distress. Her PaO2 is 42 mmHg and FIO2 is 80%. Which intervention should the nurse complete first? A Stat portable chest radiography B Administer lorazepam (Ativan) 1 mg IV push C Place the patient in a prone position on a rotational bed D Position the patient with arms supported away from the chest

D Position the patient with arms supported away from the chest (The nurse will first position the patient to facilitate ventilation. Additional oxygen support may be necessary. Refractory hypoxemia indicates the patient is not demonstrating acute lung injury but has now developed acute respiratory distress syndrome (ARDS). If the PaO2 is 42 mm Hg on 80% FIO2 (fraction of inspired oxygen; room air is 21% FIO2), then the PaO2/FIO2 ratio is 52.5, indicating ARDS (PaO2/FIO2 ratio < 200). Stat portable chest radiography may show worsening infiltrates or "white lung." A rotational bed placing the patient in prone position would be a strategy to use for select patients with ARDS. This patient's age, diagnosis, and comorbidities may indicate appropriateness for this treatment. Administration of lorazepam (Ativan) 1 mg may be harmful to this patient's oxygenation status. Further assessment would be needed to determine safety.)

A nurse prepares to discharge an older adult client home from the emergency department (ED). Which actions should the nurse take to prevent future ED visits? (SATA) a. Provide medical supplies to the family. b. Consult a home health agency. c. Encourage participation in community activities. d. Screen for depression and suicide. e. Complete a functional assessment.

D, E Due to the high rate of suicide among older adults, a nurse should assess all older adults for depression and suicide. The nurse should also screen older adults for functional assessment, cognitive assessment, and risk for falls to prevent future ED visits.

Which problem places a person at highest risk for septic shock? A. Kidney failure B. Cirrhosis C. Lung cancer D. 40% burn injury

D. 40% burn injury The skin forms the first barrier to prevent entry of organisms into the body; this client is at very high risk for sepsis and death. Although the client with kidney failure has an increased risk for infection, his skin is intact, unlike the client with burn injury. Although the liver acts as a filter for pathogens, the client with cirrhosis has intact skin, unlike the burned client. The client with lung cancer may be at risk for increased secretions and infection, but risk is not as high as for a client with open skin.

When caring for a client with decreased level of consciousness that was admitted with shock of unknown origin, which action does the nurse take first? A. Obtain IV access and hang prescribed fluid infusions. B. Apply the automatic blood pressure cuff. C. Assess level of consciousness and pupil reaction to light. D. Check the airway and respiratory status.

D. Check the airway and respiratory status. When caring for any client, determining airway and respiratory status is the priority. The airway takes priority over obtaining IV access, applying the blood pressure cuff, and assessing for changes in the client's mental status.

Which nurse should be assigned to care for an intubated client who has septic shock as the result of a methicillin-resistant Staphylococcus aureus (MRSA) infection? A. The LPN/LVN who has 20 years of experience B. The new RN who recently finished orienting and is working independently with moderately complex clients C. The RN who will also be caring for a client who had coronary artery bypass graft (CABG) surgery 12 hours ago D. The RN with 2 years of experience in intensive care

D. The RN with 2 years of experience in intensive care The RN with current intensive care experience who is not caring for a postoperative client would be an appropriate assignment. Care of the unstable client with intubation and mechanical ventilation is not within the scope of practice for the LPN/LVN. A client who is experiencing septic shock is too complex for the new RN. Although the RN who is also caring for the post-CABG client is experienced, this assignment will put the post-CABG client at risk for MRSA infection.

The nurse is caring for postoperative clients at risk for hypovolemic shock. Which condition represents an early symptom of shock? A. Hypotension B. Bradypnea C. Heart blocks D. Tachycardia

D. tachycardia Heart and respiratory rates increased from the client's baseline level or a slight increase in diastolic blood pressure may be the only objective manifestation of this early stage of shock. Catecholamine release occurs early in shock as a compensation for fluid loss; blood pressure will be normal. Early in shock, the client displays rapid, not slow, respirations. Dysrhythmias are a late sign of shock; they are related to lack of oxygen to the heart.

1. When assisting with oral intubation of a patient who is having respiratory distress, in which order will the nurse take these actions? (Put a comma and a space between each answer choice [A, B, C, D, E].) a. Obtain a portable chest-x-ray. b. Position the patient in the supine position. c. Inflate the cuff of the endotracheal tube after insertion. d. Attach an end-tidal CO2 detector to the endotracheal tube. e. Oxygenate the patient with a bag-valve-mask device for several minutes.

E, B, C, D, A

Which signs and symptoms differentiate hypoxemic respiratory failure from hypercapnic respiratory failure (select all that apply)? a. Cyanosis b. Tachypnea c. Morning headache d. Paradoxic breathing e. Use of pursed-lip breathing

a, b, d (Clinical manifestations that occur with hypoxemic respiratory failure include cyanosis, tachypnea, and paradoxic chest or abdominal wall movement with the respiratory cycle. Clinical manifestations of hypercapnic respiratory failure include morning headache, pursed-lip breathing, and decreased or increased respiratory rate with shallow breathing.)

Patients with acute respiratory failure will have drug therapy to meet their individual needs. Which drugs will meet the goal of reducing pulmonary congestion (select all that apply)? a. Morphine b. Ceftriaxone (Rocephin) c. Nitroglycerin (Tridil) d. Furosemide (Lasix) e. Albuterol (Ventolin) f. Methylprednisolone (Solu-Medrol)

a, c, d (Morphine and nitroglycerin will decrease pulmonary congestion caused by heart failure; IV diuretics (e.g., furosemide [Lasix]) are also used. Inhaled albuterol (Ventolin) or metaproterenol (Alupent) will relieve bronchospasms. Ceftriaxone (Rocephin) and azithromycin (Zithromax) are used to treat pulmonary infections. Methylprednisolone (Solu-Medrol), an IV corticosteroid, will reduce airway inflammation. Morphine is also used to decrease anxiety, agitation, and pain.)

Which changes of aging contribute to the increased risk for respiratory failure in older adults (select all that apply)? a. Alveolar dilation b. Increased delirium muscle strength c. Changes in vital signs d. Increased infection risk e. Decreased respiratory f. Diminished elastic recoil within the airways

a, d, e, f (Changes from aging that increase the older adult's risk for respiratory failure include alveolar dilation, increased risk for infection, decreased respiratory muscle strength, and diminished elastic recoil in the airways. Although delirium can complicate ventilator management, it does not increase the older patient's risk for respiratory failure. The older adult's blood pressure (BP) and heart rate (HR) increase but this does not affect the risk for respiratory failure. The ventilatory capacity is decreased and the larger air spaces decrease the surface area for gas exchange, which increases the risk.)

The patient progressed from acute lung injury to acute respiratory distress syndrome (ARDS). He is on the ventilator & receiving propofol(Diprivan) for sedation & fentanyl (Sublimaze) to decrease anxiety, agitation, & pain in order to decrease his work of breathing, O2 consumption, CO2 production & risk of injury. What intervention is recommended in caring for this patient? a. A sedation holiday b. Monitoring for hypermetabolism c. Keeping his legs still to avoid dislodging the airway d. Repositioning him every 4 hours to decrease agitation

a. A sedation holiday (A sedation holiday is needed to assess the patient's condition and readiness to extubate. A hypermetabolic state occurs with critical illness. With malnourished patients, enteral or parenteral nutrition is started within 24 hours; with well-nourished patients it is started within 3 days. With these medications, the patient will be assessed for cardiopulmonary depression. Venous thromboembolism prophylaxis will be used but there is no reason to keep the legs still. Repositioning the patient every 2 hours may help to decrease discomfort and agitation)

Priority Decision: After endotracheal intubation and mechanical ventilation have been started, a patient in respiratory failure becomes very agitated and is breathing asynchronously with the ventilator. What is it most important for the nurse to do first? a. Evaluate the patient's pain level, ABGs, and electrolyte values b. Sedate the patient to unconsciousness to eliminate patient awareness c. Administer the PRN vecuronium (Norcuron) to promote synchronous ventilations d. Slow the rate of ventilations provided by the ventilator to allow for spontaneous breathing by the patient

a. Evaluate the patient's pain level, ABGs, and electrolyte values (It is most important to assess the patient for the cause of the restlessness and agitation (e.g., pain, hypoxemia, electrolyte imbalances) and treat the underlying cause before sedating the patient. Although sedation, analgesia, and neuromuscular blockade are often used to control agitation and pain, these treatments may contribute to prolonged ventilator support and hospital days.)

A patient with a massive hemothorax and pneumothorax has absent breath sounds in the right lung. To promote improved V/Q matching, how should the nurse position the patient? a. On the left side b. On the right side c. In a reclining chair bed d. Supine with the head of the bed elevated

a. On the left side (When there is impaired function of one lung, the patient should be positioned with the unaffected lung in the dependent position to promote perfusion to the functioning tissue. If the diseased lung is positioned dependently, more V/Q mismatch would occur. The head of the bed may be elevated or a reclining chair may be used, with the patient positioned on the unaffected side, to maximize thoracic expansion if the patient has increased work of breathing.)

A patient has a PaO2 of 50 mm Hg and a PaCO2 of 42 mm Hg because of an intrapulmonary shunt. Which therapy is the patient most likely to respond best to? a. Positive pressure ventilation b. Oxygen administration at a FIO2 of 100% c. Administration of O2 per nasal cannula at 1 to 3 L/min d. Clearance of airway secretions with coughing and suctioning

a. Positive pressure ventilation (Patients with a shunt are usually more hypoxemic than patients with a V/Q mismatch because the alveoli are filled with fluid, which prevents gas exchange. Hypoxemia resulting from an intrapulmonary shunt is usually not responsive to high O2 concentrations and the patient will usually require positive pressure ventilation. Hypoxemia associated with a V/Q mismatch usually responds favorably to O2 administration at 1 to 3 L/min by nasal cannula. Removal of secretions with coughing and suctioning is generally not effective in reversing an acute hypoxemia resulting from a shunt.)

When mechanical ventilation is used for the patient with ARDS, what is the rationale for applying positive end-expiratory pressure (PEEP)? a. Prevent alveolar collapse and open up collapsed alveoli b. Permit smaller tidal volumes with permissive hypercapnia c. Promote complete emptying of the lungs during exhalation d. Permit extracorporeal oxygenation and carbon dioxide removal outside the body

a. Prevent alveolar collapse and open up collapsed alveoli (Positive end-expiratory pressure (PEEP) used with mechanical ventilation applies positive pressure to the airway and lungs at the end of exhalation, keeping the lung partially expanded and preventing collapse of the alveoli and helping to open up collapsed alveoli. Permissive hypercapnia is allowed when the patient with ARDS is ventilated with smaller tidal volumes to prevent barotrauma. Extracorporeal membrane oxygenation and extracorporeal CO2 removal involve passing blood across a gas-exchanging membrane outside the body and then returning oxygenated blood to the body.)

In caring for the patient with ARDS, what is the most characteristic sign the nurse would expect the patient to exhibit? a. Refractory hypoxemia b. Bronchial breath sounds c. Progressive hypercapnia d. Increased pulmonary artery wedge pressure (PAWP)

a. Refractory hypoxemia (Refractory hypoxemia, hypoxemia that does not respond to increasing concentrations of oxygenation by any route, is a hallmark of ARDS and is always present. Bronchial breath sounds may be associated with the progression of ARDS. PaCO2 levels may be normal until the patient is no longer able to compensate in response to the hypoxemia. Pulmonary artery wedge pressure (PAWP) that is normally elevated in cardiogenic pulmonary edema is normal in the pulmonary edema of ARDS.)

Which assessment finding should cause the nurse to suspect the early onset of hypoxemia? a. Restlessness b. Hypotension c. Central cyanosis d. Cardiac dysrhythmias

a. Restlessness (Because the brain is very sensitive to a decrease in oxygen delivery, restlessness, agitation, disorientation, and confusion are early signs of hypoxemia, for which the nurse should be alert. Mild hypertension is also an early sign, accompanied by tachycardia. Central cyanosis is an unreliable, late sign of hypoxemia. Cardiac dysrhythmias also occur later)

The best patient response to treatment of ARDS occurs when initial management includes what? a. Treatment of the underlying condition b. Administration of prophylactic antibiotics c. Treatment with diuretics and mild fluid restriction d. Endotracheal intubation and mechanical ventilation

a. Treatment of the underlying condition (Because ARDS is precipitated by a physiologic insult, a critical factor in its prevention and early management is treatment of the underlying condition. Prophylactic antibiotics, treatment with diuretics and fluid restriction, and mechanical ventilation are also used as ARDS progresses.)

The most common early clinical manifestations of ARDS that the nurse may observe are a. dyspnea and tachypnea. b. cyanosis and apprehension. c. hypotension and tachycardia. d. respiratory distress and frothy sputum.

a. dyspnea and tachypnea. (The initial presentation of acute respiratory distress syndrome (ARDS) is often subtle. At the time of the initial injury and for several hours up to 2 days afterward, the patient may not experience respiratory symptoms, or the patient may exhibit only dyspnea, tachypnea, cough, and restlessness.)

Although ARDS may result from direct lung injury or indirect lung injury as a result of systemic inflammatory response syndrome (SIRS), the nurse is aware that ARDS is most likely to occur in the patient with a host insult resulting from a. sepsis. b. oxygen toxicity c. prolonged hypotension. d. cardiopulmonary bypass.

a. sepsis. (Although ARDS may occur in the patient who has virtually any severe illness and may be both a cause and a result of systemic inflammatory response syndrome (SIRS), the most common precipitating insults of ARDS are sepsis, gastric aspiration, and severe massive trauma.)

Which descriptions are characteristic of hypoxemic respiratory failure (SATA)? a. Referred to as ventilatory failure b. Primary problem is inadequate O2 transfer c. Risk of inadequate O2 saturation of hemoglobin exists d. Body is unable to compensate for acidemia of increased PaCO2 e. Most often caused by ventilation-perfusion (V/Q) mismatch and shunt f. Exists when PaO2 is 60 mm Hg or less, even when O2 is administered at 60%

b, c, e, f (Hypoxemic respiratory failure is often caused by ventilation-perfusion (V/Q) mismatch & shunt. It is called oxygenation failure because the primary problem is inadequate oxygen transfer. There is a risk of inadequate oxygen saturation of hemoglobin and it exists when PaO2 is 60 mm Hg or less, even when oxygen is administered at 60%. Ventilatory failure is hypercapnic respiratory failure. Hypercapnic respiratory failure results from an imbalance between ventilatory supply & ventilatory demand & the body is unable to compensate for the acidemia of increased PaCO2)

The nurse suspects that a patient with PEEP is experiencing negative effects of this ventilatory maneuver when which of the following is assessed? a. Increasing PaO2 b. Decreasing blood pressure c. Decreasing heart rate (HR) d. Increasing central venous pressure (CVP)

b. Decreasing blood pressure (PEEP increases intrathoracic and intrapulmonic pressures, compresses the pulmonary capillary bed, and reduces blood return to both the right and left sides of the heart. Increased PaO2 is an expected effect of PEEP. Preload (CVP) and cardiac output (CO) are decreased, often with a dramatic decrease in BP.)

Which physiologic mechanism of hypoxemia occurs with pulmonary fibrosis? a. Anatomic shunt b. Diffusion limitation c. Intrapulmonary shunt d. V/Q mismatch ratio of less than 1

b. Diffusion limitation (Diffusion limitation in pulmonary fibrosis is caused by thickened alveolar-capillary interface, which slows gas transport.)

A patient in hypercapnic respiratory failure has a nursing diagnosis of ineffective airway clearance related to increasing exhaustion. What is an appropriate nursing intervention for this patient? a. Inserting an oral airway b. Performing augmented coughing c. Teaching the patient huff coughing d. Teaching the patient slow pursed lip breathing

b. Performing augmented coughing (Augmented coughing is done by applying pressure on the abdominal muscles at the beginning of expiration. This type of coughing helps to increase abdominal pressure and expiratory flow to assist the cough to remove secretions in the patient who is exhausted. An oral airway is used only if there is a possibility that the tongue will obstruct the airway. Huff coughing prevents the glottis from closing during the cough & works well for patients with COPD to clear central airways. Slow pursed lip breathing allows more time for expiration and prevents small bronchioles from collapsing.)

A patient with ARDS has a nursing diagnosis of risk for infection. To detect the presence of infections commonly associated with ARDS, what should the nurse monitor? a. Gastric aspirate for pH and blood b. Quality, quantity, and consistency of sputum c. Subcutaneous emphysema of the face, neck, and chest d. Mucous membranes of the oral cavity for open lesions

b. Quality, quantity, and consistency of sputum (Ventilator-associated pneumonia (VAP) is one of the most common complications of ARDS. Early detection requires frequent monitoring of sputum smears & cultures & assessment of the quality, quantity, & consistency of sputum. Prevention of VAP is done with strict infection control measures, ventilator bundle protocol, and subglottal secretion drainage. Blood in gastric aspirate may indicate a stress ulcer and subcutaneous emphysema of the face, neck, and chest occurs with barotrauma during mechanical ventilation. Oral infections may result from prophylactic antibiotics & impaired host defenses but are not common.)

Which patient with the following manifestations is most likely to develop hypercapnic respiratory failure? a. Rapid, deep respirations in response to pneumonia b. Slow, shallow respirations as a result of sedative overdose c. Large airway resistance as a result of severe bronchospasm d. Poorly ventilated areas of the lung caused by pulmonary edema

b. Slow, shallow respirations as a result of sedative overdose (Hypercapnic respiratory failure is associated with alveolar hypoventilation with increases in alveolar and arterial carbon dioxide (CO2) & often is caused by problems outside the lungs. A patient with slow, shallow respirations is not exchanging enough gas volume to eliminate CO2. Deep, rapid respirations reflect hyperventilation and often accompany lung problems that cause hypoxemic respiratory failure. Pulmonary edema and large airway resistance cause obstruction of oxygenation & result in a V/Q mismatch or shunt typical of hypoxemic respiratory failure.

The patient is being admitted to the intensive care unit (ICU) with hypercapnic respiratory failure. Which manifestations should the nurse expect to assess in the patient (select all that apply)? a. Cyanosis b. Metabolic acidosis c. Morning headache d. Respiratory acidosis e. Use of tripod position f. Rapid, shallow respirations

c, d, e, f (Morning headache, respiratory acidosis, the use of tripod position, and rapid, shallow respirations would be expected. The other manifestations are characteristic of hypoxemic respiratory failure.)

In patients with ARDS who survive the acute phase of lung injury, what manifestations are seen when they progress to the fibrotic phase? a. Chronic pulmonary edema and atelectasis b. Resolution of edema and healing of lung tissue c. Continued hypoxemia because of diffusion limitation d. Increased lung compliance caused by the breakdown of fibrotic tissue

c. Continued hypoxemia because of diffusion limitation (In the fibrotic phase of ARDS, diffuse scarring and fibrosis of the lungs occur, resulting in decreased surface area for gas exchange and continued hypoxemia caused by diffusion limitation. Although edema is resolved, lung compliance is decreased because of interstitial fibrosis. Long-term mechanical ventilation is required. The patient has a poor prognosis for survival.)

The nurse suspects the early stage of ARDS in any seriously ill patient who manifests what? a. Develops respiratory acidosis b. Has diffuse crackles and rhonchi c. Exhibits dyspnea and restlessness d. Has a decreased PaO2 and an increased PaCO2

c. Exhibits dyspnea and restlessness (Early signs of ARDS are insidious and difficult to detect but the nurse should be alert for any early signs of hypoxemia, such as dyspnea, restlessness, tachypnea, cough, and decreased mentation, in patients at risk for ARDS. Abnormal findings on physical examination or diagnostic studies, such as adventitious lung sounds, signs of respiratory distress, respiratory alkalosis, or decreasing PaO2, are usually indications that ARDS has progressed beyond the initial stages.)

When explaining respiratory failure to the patient's family, what should the nurse use as an accurate description? a. The absence of ventilation b. Any episode in which part of the airway is obstructed c. Inadequate gas exchange to meet the metabolic needs of the body d. An episode of acute hypoxemia caused by a pulmonary dysfunction

c. Inadequate gas exchange to meet the metabolic needs of the body (Respiratory failure results when the transfer of oxygen or carbon dioxide function of the respiratory system is impaired and, although the definition is determined by PaO2 and PaCO2 levels, the major factor in respiratory failure is inadequate gas exchange to meet tissue oxygen (O2) needs. Absence of ventilation is respiratory arrest and partial airway obstruction may not necessarily cause respiratory failure. Acute hypoxemia may be caused by factors other than pulmonary dysfunction)

The patient with a history of heart failure and acute respiratory failure has thick secretions that she is having difficulty coughing up. Which intervention would best help to mobilize her secretions? a. Administer more IV fluid b. Perform postural drainage c. Provide O2 by aerosol mask d. Suction airways nasopharyngeally

c. Provide O2 by aerosol mask (For the patient with a history of heart failure, current acute respiratory failure, and thick secretions, the best intervention is to liquefy the secretions with either aerosol mask or using normal saline administered by a nebulizer. Excess IV fluid may cause cardiovascular distress and the patient probably would not tolerate postural drainage with her history. Suctioning thick secretions without thinning them is difficult and increases the patient's difficulty in maintaining oxygenation. With copious secretions, this could be done after thinning the secretions.)

When the V/Q lung scan result returns with a mismatch ratio that is greater than 1, which condition should be suspected? a. Pain b. Atelectasis c. Pulmonary embolus d. Ventricular septal defect

c. Pulmonary embolus (There will be more ventilation than perfusion (V/Q ratio greater than 1) with a pulmonary embolus. Pain and atelectasis will cause a V/Q ratio less than 1. A ventricular septal defect causes an anatomic shunt as the blood bypasses the lungs.)

When teaching the patient about what was happening when experiencing an intrapulmonary shunt, which explanation is accurate? a. This occurs when an obstruction impairs the flow of blood to the ventilated areas of the lung. b. This occurs when blood passes through an anatomic channel in the heart and bypasses the lungs. c. This occurs when blood flows through the capillaries in the lungs without participating in gas exchange. d. Gas exchange across the alveolar capillary interface is compromised by thickened or damaged alveolar membranes.

c. This occurs when blood flows through the capillaries in the lungs without participating in gas exchange. (Intrapulmonary shunt occurs when blood flows through the capillaries in the lungs without participating in gas exchange (e.g., acute respiratory distress syndrome [ARDS], pneumonia). Obstruction impairs the flow of blood to the ventilated areas of the lung in a V/Q mismatch ratio greater than 1 (e.g., pulmonary embolus). Blood passes through an anatomic channel in the heart and bypasses the lungs with anatomic shunt (e.g., ventricular septal defect). Gas exchange across the alveolar capillary interface is compromised by thickened or damaged alveolar membranes in diffusion limitation (e.g., pulmonary fibrosis, ARDS).)

Which intervention is most likely to prevent or limit barotrauma in the patient with ARDS who is mechanically ventilated? a. Decreasing PEEP b. Increasing the tidal volume c. Use of permissive hypercapnia d. Use of positive pressure ventilation

c. Use of permissive hypercapnia (To avoid barotrauma and minimize risk associated with elevated plateau and peak inspiratory pressures, many patient with ARDS are ventilated with smaller tidal volumes and varying amounts of positive end-expiratory pressure (PEEP) to minimize O2 requirements and intrathoracic pressures. One result of this protocol is an elevation in partial pressure of CO2 in arterial blood (PaCO2), called permissive hypercapnia because the PaCO2 is allowed to rise above normal limits.)

The nurse assesses that a patient in respiratory distress is developing respiratory fatigue and the risk of respiratory arrest when the patient displays which behavior? a. Cannot breathe unless he is sitting upright b. Uses the abdominal muscles during expiration c. Has an increased inspiratory-expiratory (I/E) ratio d. Has a change in respiratory rate from rapid to slow

d. Has a change in respiratory rate from rapid to slow (The increase in respiratory rate required to blow off accumulated CO2 predisposes to respiratory muscle fatigue. The slowing of a rapid rate in a patient in acute distress indicates tiring and the possibility of respiratory arrest unless ventilatory assistance is provided. A decreased inspiratory-expiratory (I/E) ratio, orthopnea, and accessory muscle use are common findings in respiratory distress but do not necessarily signal respiratory fatigue or arrest.)

In caring for a patient in acute respiratory failure, the nurse recognizes that noninvasive positive pressure ventilation (NIPPV) may be indicated for which patient? a. Is comatose and has high oxygen requirements b. Has copious secretions that require frequent suctioning c. Responds to hourly bronchodilator nebulization treatments d. Is alert and cooperative but has increasing respiratory exhaustion

d. Is alert and cooperative but has increasing respiratory exhaustion (Noninvasive positive pressure ventilation (NIPPV) involves the application of a face mask and delivery of a volume of air under inspiratory pressure. Because the device is worn externally, the patient must be able to cooperate in its use and frequent access to the airway for suctioning or inhaled medications must not be necessary. It is not indicated when high levels of oxygen are needed or respirations are absent.)

Which arterial blood gas (ABG) results would most likely indicate acute respiratory failure in a patient with chronic lung disease? a. PaO2 52 mm Hg, PaCO2 56 mm Hg, pH 7.4 b. PaO2 46 mm Hg, PaCO2 52 mm Hg, pH 7.36 c. PaO2 48 mm Hg, PaCO2 54 mm Hg, pH 7.38 d. PaO2 50 mm Hg, PaCO2 54 mm Hg, pH 7.28

d. PaO2 50 mm Hg, PaCO2 54 mm Hg, pH 7.28 (In a patient with normal lung function, respiratory failure is commonly defined as a PaO2 ≤60 mm Hg or a PaCO2 >45 mm Hg or both. However, because the patient with chronic pulmonary disease normally maintains low PaO2 and high PaCO2 , acute respiratory failure in these patients can be defined as an acute decrease in PaO2 or an increase in PaCO2 from the patient's baseline parameters, accompanied by an acidic pH. The pH of 7.28 reflects an acidemia and a loss of compensation in the patient with chronic lung disease.)

What is the primary reason that hemodynamic monitoring is instituted in severe respiratory failure? a. To detect V/Q mismatches b. To continuously measure the arterial BP c. To evaluate oxygenation and ventilation status d. To evaluate cardiac status and blood flow to tissues

d. To evaluate cardiac status and blood flow to tissues (Hemodynamic monitoring with a pulmonary artery catheter is instituted in severe respiratory failure to determine the amount of blood flow to tissues and the response of the lungs and heart to hypoxemia. Continuous BP monitoring may be performed but BP is a reflection of cardiac activity, which can be determined by the pulmonary artery catheter findings. Arterial blood gases (ABGs) are important to evaluate oxygenation and ventilation status and V/Q mismatches.)

The O2 delivery system chosen for the patient in acute respiratoryfailure should: a. always be a low-flow device, such as a nasal cannula or face mask. b. administer continuous positive airway pressure ventilation to prevent CO2 narcosis. c. correct the PaO2 to a normal level as quickly as possible using mechanical ventilation. d. maintain the PaO2 at greater than or equal to 60 mm Hg at the lowest O2 concentration possible.

d. maintain the PaO2 at greater than or equal to 60 mm Hg at the lowest O2 concentration possible. (The selected O2 delivery system must maintain partial pressure of O2 in arterial blood (PaO2) at 55 to 60 mm Hg or higher and arterial O2 saturation (SaO2) at 90% or higher at the lowest O2 concentration possible.)

Prone positioning is considered for a patient with ARDS who has not responded to other measures to increase PaO2 . The nurse knows that this strategy will a. increase the mobilization of pulmonary secretions. b. decrease the workload of the diaphragm and intercostal muscles. c. promote opening of atelectatic alveoli in the upper portion of the lung. d. promote perfusion of nonatelectatic alveoli in the anterior portion of the lung.

d. promote perfusion of nonatelectatic alveoli in the anterior portion of the lung. (When a patient with ARDS is supine, alveoli in the posterior areas of the lung are dependent & fluid-filled and the heart and mediastinal contents place more pressure on the lungs, predisposing to atelectasis. If the patient is turned prone, air-filled nonatelectatic alveoli in the anterior portion of the lung receive more blood and perfusion may be better matched to ventilation, causing less V/Q mismatch. Lateral rotation therapy is used to stimulate postural drainage and help mobilize pulmonary secretions.)


Conjuntos de estudio relacionados

General Behavioral Health Course Objectives, Diagnosis: Behavioral Health PA 604

View Set

Evolve: School-Age Childern (Lvl 3)

View Set

Principles of Microeconomics Exam1 Review Questions Ch 1-4

View Set